Download as pdf or txt
Download as pdf or txt
You are on page 1of 40

LEGALEDGE TEST SERIES

Part of the Most Comprehensive & Consistently Successful Study Material & Test Series Module, spanning across
both Offline and Online Programs in the entire Country. As a result LegalEdge was able to engineer Clean-Sweep-
Landslide figures of a handsome 64 Selections & 65 Selections in Top 100 (including AIR 1, 2 & 3 from Classroom
Contact Programs in 2022) & a whopping 273 selections & 327 selections in Top 500, in CLAT 2021 & CLAT 2022,
respectively. With AILET being no different, a total of 34 of our students found their way into NLU, Delhi in 2021 &
35 in 2022. In a nutshell, every second admit in a Top National Law School in 2021 & 2022 came from the LegalEdge
Preparation Ecosystem.

MOCK COMMON LAW ADMISSION TEST 2023


MOCK CLAT - 39

Scan this code after the test


Duration : 120 Minutes Candidate Name : _____________
Max. Marks : 150 Batch : _____________
Centre Name : __________ Contact No. : _____________

INSTRUCTIONS TO CANDIDATES to punch in your answers


(Test ID: 2585316)
1. No clarification on the question paper can be sought. Answer the questions as they are.
2. There are 150 multiple choice objective type questions.
3. There is negative marking of 0.25 for every incorrect answer. Each question carries ONE mark. Total marks are
150
4. You have to indicate the correct answer by darkening one of the four responses provided, with a BALL PEN
(BLUE OR BLACK) in the OMR Answer Sheet.
Example: For the question, "Where is the Taj Mahal located?", the correct answer is (b).
The student has to darken the corresponding circle as indicated below:
(a) Kolkata (b) Agra (c) Bhopal (d) Delhi
Right Method Wrong Methods

5. Answering the questions by any method other than the method indicated above shall be considered incorrect and
no marks will be awarded for the same.
6. More than one response to a question shall be counted as wrong.
7. Do not write anything on the OMR Answer Sheet other than the details required and, in the spaces, provided for.
8. You are not required to submit the OMR Answer Sheet and Test Paper after the test.
9. The use of any unfair means by any candidate shall result in the cancellation of his/her candidature.
10. Impersonation is an offence and the student, apart from disqualification, may have to face criminal prosecution.
11. You have to scan the QR code only after completion of offline test.
12. You cannot leave the examination hall without punching your answers on the portal.
SECTION-A: ENGLISH LANGUAGE

Directions (Q.1-Q.30): Read the following passage carefully and answer the questions that follow.

Passage (Q.1 – Q.5): On 15 August 2021, the Taliban swept back to power in Kabul. This was less than a month
short of the 20th anniversary of the terrorist attacks of 11 September 2001 that launched the global ‘war on terror’
and precipitated the ouster of the Taliban from power in Afghanistan, in retaliation for their hosting of the Al-
Qaeda perpetrators of the attack.

On 7 September 2021, the Taliban announced their caretaker government, sporting a prime minister on the UN’s
sanctions list and an interior minister on the US’ terrorist list, and not a single woman. This came one week
before the UN International Day of Democracy celebrated on 15 September each year.
It is in this sobering context that the author seeks to address the strained nexus between terrorism, security and
democracy that has often raised its head in the last twenty years of the ‘war on terror’ and deserves renewed
scrutiny today. [1] In effect, the relationship between the three keywords that have dominated public discourse
and in turn impelled or paralysed political decisions since the calamitous events of 9/11 has become increasingly
complex over the past two decades. These keywords—democracy, security and terrorism—are not new for they
lie at the core of all debates on democracy since its early infancy in Mesopotamia, Athens and elsewhere. While
democracy and security have frequently been challenged through the centuries by extremists, secessionists,
terrorists, or other such threats, never have their relationships with each other been more vexed and in need of
critical self-examination and honest redress than today.

"In the fight against terrorism, democracies should not only pursue state or homeland security but human
security." At this twentieth anniversary of the ‘war on terror’, despite the innumerable scholarly, political and
public debates fostered by and since 9/11, democracy continues to face particular threats. These threats are
threefold.
The first threat is the obvious one that Al-Qaeda-inspired terrorism undermines democracy both deliberately and
indirectly and attacks the security of citizens which is democracy’s central asset.
The second is that the attempt to prevent terrorism through democracy promotion, a deliberate strategy of the
‘war on terror’, has backfired into a broad backlash against democracy promotion initiatives and democracy
itself.

Third, the pursuit of the ‘war on terror’ itself poses a significant threat to democracy by eroding the core values
of democracy, namely human rights, rule of law and legitimacy.
In effect, both terrorism and the war on terrorism have upset the delicate balance between democracy and security
and placed democracy at risk.
As we mark sombrely the 20th anniversary, it is more urgent than ever to recognise and redress these three threats
and the ways in which they have endangered the fragile balance between security and democracy and initiated
an insidious process of eroding democracy itself.

1. Which option, if true, weakens the argument of the author that the Taliban is a threat to democracy?
(a) The Taliban government now features women in their cabinet.
(b) The Taliban’s values have shifted greatly in the last twenty years for the better.
(c) The Taliban will continue to uphold what it believes and go to any lengths for that.
(d) The Taliban now vouches to engage with other countries.

2. The author of the passage has not mentioned which of the following in the passage?
(a) The nexus between terrorism, democracy and security.
(b) Challenges of the threats to democracy from terrorism.
(c) Counters to the challenges of the threats to democracy.
(d) Taliban’s return to power in Afghanistan.

Head Office: 127, Zone II, MP Nagar, Bhopal |+91-7676564400| https://www.toprankers.com Page 2 of 40
3. Out of the following, which option is most likely to be true? Use the information in the passage.
(a) Democracy is vulnerable to erosion if the threats to it are not managed.
(b) Pursuing state or homeland security is paramount in the fight against terrorism.
(c) Public or political debates on the 9/11 attacks are considered inappropriate.
(d) Democracy, security and terrorism may have a common nexus except for their basics.

4. The sentence marked as [1] in the passage is:


(a) Incorrect; it should read: “In affect, the relationship between the three keywords that have dominated public
discourse and in turn impelled or paralysed political decisions since the calamitous events of 9/11 has become
increasingly complex over the past two decades.”
(b) Incorrect; it should read: “In effect, the relationship between the three keywords that have dominated public
discourse and in turn impelled or paralysed political decisions since the calamitous events of 9/11 has become
increasingly complex on the past two decades.”
(c) Incorrect; it should read: “In effect, the relationships between the three keywords that have dominated public
discourse and in turn impelled or paralysed political decisions since the calamitous events of 9/11 has become
increasingly complex over the past two decades.”
(d) Correct. The sentence does not need correction.

5. Which among the following is correct about the present Taliban government in Afghanistan?
(a) The present government is capable of handling the threats to democracy.
(b) The present government has adequate representation from all sections of society.
(c) The present government is likely a temporary government and is subject to change.
(d) The present government has all its basics covered to govern the country.

Passage (Q.6 – Q.10): Authoritarianism is anathema to democracy. For a pluralistic and open society like ours,
democracy is the best suited model. Any institution that tends to deviate from the core values of respecting
individual freedom, dignity and cultural and social diversity, faces the risk of losing public faith. Chief Justice
of India NV Ramana's recent observations on the functioning of the police in general and the CBI in particular
warrant serious consideration and a wider national debate. Lamenting that some of the actions and inactions of
the CBI have raised questions regarding its credibility and that people hesitate to approach the police in times of
despair, the CJI made a strong pitch for police reforms, an idea that has been hanging fire for decades.

Unfortunately, the image of the police as an institution is often tarnished by allegations of corruption, excesses,
lack of impartiality and close nexus with the political class. Instead of inspiring confidence, the police system
typically evokes fear of harassment among the general public. It is a legacy problem inherited from the colonial
rulers who had used policing as a tool to instil fear among people. This must change, and the change should start
at the training stage itself. There have been several recommendations by the expert committees in the past for
improving efficiency and professionalism in the police system, but the situation on the ground is far from
encouraging.

Lack of adequate infrastructure and manpower, pathetic working conditions, especially at the lowest rung, lack
of modern equipment, questionable methods of procuring evidence, officers failing to abide by the rule book and
the lack of accountability of erring officers are some of the key issues adversely affecting the system. Reforming
the police administration is long overdue, a fact acknowledged by the Union Home Ministry in a recent status
note.

The CJI has rightly called for the creation of an independent and autonomous investigative agency to insulate it
from political influences. There is also a strong case for the establishment of an independent umbrella institution
to bring various agencies like the CBI, Serious Fraud Investigation Office (SFIO), and the Enforcement
Directorate (ED) under one roof. This body is required to be created under a statute, clearly defining its powers,

Head Office: 127, Zone II, MP Nagar, Bhopal |+91-7676564400| https://www.toprankers.com Page 3 of 40
functions and jurisdictions. Such a law is expected to lead to a much-needed legislative oversight. This umbrella
organisation will help put an end to the multiplicity of proceedings.

The proposed central law for the umbrella investigative body can be replicated by the States as well because it
is the State level investigating agencies which handle most of the cases and they must enjoy the same level of
credibility as that of the national agency. Since the police have an active interface with the people, there is a need
to re-orient their training modules to reflect the changing needs and sensitise them about the people's concerns.

6. Which of the following words is closest to the word 'anathema'?


(a) Love (b) Opposite (c) Treasure (d) Curse

7. According to the passage, what could be the characteristics of authoritarianism?


(a) An Inhibition of individual freedom, dignity and cultural and social diversity.
(b) Corruption, harassment and excesses by the police force.
(c) An independent and autonomous investigative agency.
(d) Both a and b.

8. What is the tone of the passage?


(a) Belligerent (b) Indignant (c) Commiserating (d) Dogmatic

9. Which of the following options uses the idiom 'hanging fire' correctly?
(a) The construction of the dam was hanging fire because of a lack of funds.
(b) Ronald said that he was hanging fire on the agreement that Stef had proposed.
(c) The issue of slavery is hanging fire for a very long time.
(d) Rahul was hanging fire for not getting the job.

10. What could be the most logical purpose of this article?


(a) Blame the government for the current state of the police department.
(b) Apprise the readers of the current state of the police department and suggest reforms.
(c) Request the judiciary to take some action against the police department.
(d) Define Authoritarianism through the actions of the police department.

Passage (Q.11 – Q.15): Paragraph 1: Educational success today has boiled down to a number game. How well
you score in the Board and entrance exams ensure admission into the college of your choice. If you wish to
pursue a mainline, stereotypical career, then being a ‘good student’ is simply not good enough. You have to be
one of the best.

Paragraph 2: But not all are equally gifted. There will always be achievers and failures. Ever wondered how it
goes for the ones in between? Those who didn’t score high yet haven’t failed. Those classified as ‘average
students’ or ‘low-scorers’. For these students, the pressure to succeed and consequent failure to achieve the
requisite grades causes anguish and despair. They feel treated as non-entities because, in our education system,
undue importance is given to high scorers. They need not lose hope however and must understand that numerous
career avenues are open for them, regardless of their scores in the Board exams.

Paragraph 3: Psychologist Swathi Menon insists that low scores are not the end of the world. ‘There are so many
avenues to explore. Most importantly, avoid making hasty decisions. Don’t panic and do not choose an unsuitable
vocation. Take time off to think right. You need to acknowledge your negative feelings and vent these off. A
problem shared is a problem halved. Look up vocational courses designed to provide you with practical skills
for work that could make you highly employable. Crisis or disappointments are a part of life. You need to deal
with these effectively to move on,” she advises.

Head Office: 127, Zone II, MP Nagar, Bhopal |+91-7676564400| https://www.toprankers.com Page 4 of 40
Paragraph 4: Another reality that needs correction is that most students wait until the end of their basic education
before deciding upon a career option. Career planning is a continuous process and should ideally start at the
school level itself. Deciding a future career is not easy, but the task becomes even more difficult when you’re
not a high scorer and lack information about the various available options. Says Neena Gandhi, an educational
consultant who heads a counselling and remedial centre. There’s tremendous scope because the global job market
is expanding and offers everyone, immaterial of the score-card, respectable and lucrative career opportunities in
broadly four sectors: service, vocational, skilled and creative fields.”

Paragraph 5: The job market absorbs students ranging from achievers to low-scorers – there are countless options
to choose from, including aviation, shipping, telecom, media, NGOs, BPOs, IT, Insurance, paramedical fields
like physiotherapy, optometry, x-ray technicians sericulture, dairy management, sports management, horticulture
and agro, etc. Those with an inclination for vocational/skilled work could pursue various polytechnic courses
like diplomas in draftsmanship, diploma in taxation, diploma in labour practice, company secretary etc. Creative
fields do not demand high scholastic abilities – an average student with proficiency in the required areas is the
need of the hour. Adds Gandhi, “In most fields, the ground requirement is not high scores but, the right attitude,
creativity and the zeal to learn and work hard. Proficiency in areas like communication skills also helps.”

Paragraph 6: Career counsellor and educational consultant Vivek Shukla insists that students score low because
our syllabus is weak in logic, reasoning and practical general knowledge. We need to improve the quality. Many
students who score well in Board exams end up failing the national entrance tests, and those who do well in these
tend to be average scorers in the Boards.”

11. What is the likely reason behind a general insistence that you have to be one of the best?
(a) Educational success today has been reduced to merely a number game.
(b) To make sure that one gets admission into the college of one’s choice.
(c) It is one of the parameters to distinguish a ‘good student’ from an ‘average student’.
(d) If one wishes to pursue a mainline, stereotypical course, being only a ‘good student’ is of no value.

12. Which of the following options is not the author’s viewpoint, as mentioned in the passage?
(a) Our education system is at fault as it gives undue importance to high and top scorers.
(b) Most students wait until the completion of their basic education before they make a career choice.
(c) There’s tremendous scope because the global job market is expanding and offers good career options.
(d) Creative fields do not necessitate high scholarly abilities and offer a chance to the average student.

13. From the given options, choose an appropriate title for the passage.
(a) Our Education system and its likely faults.
(b) Learning to differentiate between a ‘good student’ & ‘average student’.
(c) Making the right career choice.
(d) The importance of career counsellor and educational consultant.

14. Which of the following options doesn’t find a mention as one of the career options as enumerated in the passage?
(a) Aviation & shipping. (b) Horticulture & Agro.
(c) Company Secretary. (d) Genetic Engineering.

15. According to the passage, what is a probable reason for students scoring well in Board exams but failing the
national entrance tests?
(a) The tendency of the students to panic and make hasty decisions.
(b) Lack of quality in the syllabus, which is weak in logic, reasoning and practical general knowledge.
(c) Lack of will to improve teaching quality by extensive training of teaching staff.
(d) Outdated concepts and archaic subject matters that are not career oriented.

Head Office: 127, Zone II, MP Nagar, Bhopal |+91-7676564400| https://www.toprankers.com Page 5 of 40
Passage (Q.16 – Q.20): Freedom of religion encompasses all religions acting within the law in a particular
region, whether or not an individual religion accepts that other religions are legitimate or that freedom of
religious choice and religious plurality in general are good things. Exclusivist religions teach that theirs is the
only way to salvation and to religious truth, and some of them would even argue that it is necessary to suppress
the falsehoods taught by other religions. Some Protestant sects argue fiercely against Roman Catholicism, and
fundamentalist Christians of all kinds teach that religious practices like those of paganism and witchcraft are
pernicious. This was a common historical attitude prior to the Enlightenment and has appeared as governmental
policy into the present day under systems like Afghanistan's Taliban regime, which destroyed the ancient
Buddhas of Bamyan. Many religious believers believe that religious pluralism should entail not a competition
but cooperation, and argue that societal and theological change is necessary to overcome religious differences
between different religions, and denominational conflicts within the same religion. For most religious traditions,
this attitude is essentially based on a non-literal view of one's religious traditions, hence allowing for respect to
be engendered between different traditions on fundamental principles rather than more marginal issues. It is
perhaps summarized as an attitude which rejects focus on immaterial differences and instead gives respect to
those beliefs held in common. Giving one religion or denomination special rights that are denied to others can
weaken religious pluralism. This situation obtains in certain European countries, where Roman Catholicism or
regional forms of Protestantism have special status. Relativism, the belief that all religions are equal in their
value and that none of the religions gives access to absolute truth, is an extreme form of inclusivism. Likewise,
syncretism, the attempt to take over creeds of practices from other religions or even to blend practices or creeds
from different religions into one new faith is an extreme form of inter-religious dialogue. Syncretism must not
be confused with ecumenism, the attempt to bring closer and eventually reunite different denominations of one
religion that have a common origin but were separated by a schism. The existence of religious pluralism depends
on the existence of freedom of religion. Freedom of religion exists when different religions of a particular region
possess the same rights of worship and public expression. Freedom of religion is restrained in many Islamic
countries, such as in Saudi Arabia, where the public practice of religions other than Islam is forbidden; in Iran,
where the Baha'is have no legal rights and are persecuted, and in the Palestinian Authority, where Arab Christians
report they are frequent victims of religious persecution by Muslims. Religious freedom did not exist at all in
many Communist countries such as Albania and the Stalinist Soviet Union, where the state prevented the public
expression of religious belief and even persecuted some or all religions. This situation persists still today in North
Korea, and to some extent, in China and Vietnam.

16. According to the author, what is the basic governing principle of the freedom of religion?
(a) The freedom that people of a certain religion have to choose the region of inhabitation.
(b) The legitimacy of various religions and their acceptance of other religions.
(c) The co-existence of a number of religions acting within the law in a particular region.
(d) The religions of a particular region possess different rights of worship and public expression.

17. The non-literal view of one’s religious traditions is based on:


(a) The competition between the different religions and religious beliefs.
(b) The historical attitude responsible for destroying the Buddha statues in Bamyan.
(c) The special rights assigned to a particular religion over other religions.
(d) The attitude that focuses on common beliefs rather than the immaterial differences between religions.

18. How does the author reinforce his opinion about the weakening of religious pluralism?
(a) The author’s example of the destruction of the Buddha statues at Bamyan and the reason behind their
destruction.
(b) The author’s statement about giving special rights to a religion and denying other religions of those rights
which can lead to a weakening of religious pluralism.
(c) The example of European countries where Roman Catholicism or regional forms of Protestantism have
special status.
(d) The author mentions the need for cooperation instead of competition for freedom of religion.

Head Office: 127, Zone II, MP Nagar, Bhopal |+91-7676564400| https://www.toprankers.com Page 6 of 40
19. What is the basic difference between ecumenism and syncretism?
(a) Ecumenism is the attempt to reunite different denominations of common origin, and syncretism is the attempt
to reunite two religions of the same region.
(b) Ecumenism is the attempt to reunite different denominations of common origin, and syncretism is the belief
that all religions are equal.
(c) Ecumenism is the attempt to reunite different denominations of common origin, and syncretism is the attempt
to unite different ecumenist religions.
(d) Ecumenism is the attempt to reunite different denominations of common origin, and syncretism is the attempt
to blend practices of different religions and faith into one new faith.

20. Which of the following is supported by the passage?


A. Afghanistan's Taliban regime follows the beliefs of an exclusivist religion.
B. Syncretism exhibits the blending of two or more religious belief systems into a new system.
C. The common historical attitude before the Enlightenment rejects focus on immaterial differences.
(a) A only (b) A and B (c) B only (d) A, B and C

Passage (Q.21 – Q.24): Adult education has grown in importance in modern times, partly as industrialization
required more skilled workers, as technology has advanced leading workers to continue to update their skills and
knowledge, and also due to the increasing consciousness of the rights of all people to have a chance to fulfil their
potential. This brings greater happiness and satisfaction to individuals. At the same time, this benefits society as
a whole by supporting the development of each member of society in ways that allow them to contribute more
fully and effectively to their community. Adult education, therefore, like that of the youth, is vitally important
to the success of society.

The emergence of such educational programs may be understood as a response by largely western governments
to a changing world. These underlying changes are a move away from manufacturing to a services economy, the
emergence of the knowledge economy, and the decline of many traditional institutions, requiring individuals to
become more active in managing their lives.

This led to the realization that formal learning, typically concentrated in the earlier stages of life, can no longer
sustain the individual throughout their life. One of the reasons why lifelong education has become so important
is the acceleration of scientific and technological progress. Despite the increased duration of primary, secondary,
and tertiary education, the knowledge and skills acquired there are usually not sufficient for a professional career
spanning three or four decades.

21. How does Adult education benefits society?


(a) By making people realise that they could learn to read and write despite their age.
(b) By allowing adults to help children learn better.
(c) By making people able to afford the basic necessities of life.
(d) By allowing each member of society to become developed enough to contribute positively to their society.

22. What does the prevalence of adult educational programmes depict?


(a) The emergence of the traditional economy.
(b) The emergence of a developed economy.
(c) The emergence of the knowledge economy.
(d) The emergence of the manufacturing economy.

23. Unlike school education, what does adult education concentrate on?
(a) Personality development (b) Skill acquisition
(c) Training parents (d) Acquiring educational degrees

Head Office: 127, Zone II, MP Nagar, Bhopal |+91-7676564400| https://www.toprankers.com Page 7 of 40
24. “…allow them to contribute more fully and effectively to their community.”
The word ‘effectively’ is a/an
(a) Adverb (b) Noun (c) Verb (d) Adjective

Passage (Q.25 – Q.30): Former Reserve Bank of India (RBI) governor Raghuram Rajan said that consumers are
benefiting in a big way from the business of scale with several services in the new technology age coming for
free or very cheap, but it needs to be seen whether this can continue going ahead. Speaking in Davos at a session
of the World Economic Forum (WEF) annual meeting, Mr. Rajan said we benefit tremendously from the business
of scale; there are benefits of efficiency at large corporations, and consumers are gaining in a big way from low
prices. For example, Google provides a big amount of free services, said Mr. Rajan, who teaches at Chicago
University and is regarded as a global voice on economy.

Mr. Rajan said that we all know nothing is free, so it needs to be known who is paying for it when consumers
get it for free. He also wondered whether the business of scale is creating superstar firms” and who has the real
power — is it large corporations or the governments? ‘Obviously, they are making money somewhere, and we
need to know whether the two revenue sides of consumers and advertisers are comparable when it comes to data
and technology platforms” he added. Mr. Rajan said we need to think whether competition would continue in
the future.

“The concern is not so much about today but about tomorrow. We need to think whether these benefits from the
scale would continue for the consumers,” he said and added that “more than being about too big to manage, it is
about too big to control. Our corporations are becoming too big to control for our political systems”. The
panellists at the session also discussed big-ticket mergers, digital platforms and market uncertainty that are
transforming industries across the world. They also discussed how businesses are responding to this new strategic
context.

Bank of America chief Brian T. Moynihan, Google senior vice-president Ruth Porat and Blackstone group CEO
Stephen Schwarzman were among the participants. Mr. Moynihan said the banking industry in U.S. is still very
unconsolidated, though there are big firms like Bank of America. He also defended a strong regulatory
framework for the banking industry, saying one cannot take money from people without having a strong set of
regulations and market uncertainty that are transforming industries across the world. They also discussed how
businesses are responding to this new strategic context.

Asked whether data is the new oil, Mr. Porat said data is much better as it is not limited and data keeps getting
generated by all of us.

Mr. Rajan said the U.S. has always been very uncomfortable with any entity being very big, starting with banks
in the 1930s. He said Facebook and Google are being talked about a lot today, but the U.S. has a history of
proceeding against firms that became very big, and they were broken down. However, things are changing now
because of China coming into the picture.

25. Which of the following concerns did Mr. Rajan express?


(a) The corporations may be getting too big to control.
(b) The free services need to be accounted for by the companies.
(c) The existence of competition in markets.
(d) All of the above.

Head Office: 127, Zone II, MP Nagar, Bhopal |+91-7676564400| https://www.toprankers.com Page 8 of 40
26. Consider the following statements. Which of these is/are correct?
A. According to Mr. Moynihan, the banking industry in the U.S. is still very unconsolidated.
B. Raghuram Rajan defended a strong regulatory framework for the banking industry.
C. Blackstone Group CEO Stephen Schwarzman said data is much better as it is not limited anymore.
(a) Only A (b) Only C (c) Both A and B (d) Both B and C

27. Which of the following is a suitable title for the passage?


(a) Big ticket mergers.
(b) We are not ready for the Superstar firms
(c) Arm twisting by Superstar firms
(d) 'Superstar' firms giving a lot for free; what is the agenda?

28. Consider the following. Which of the pair is incorrect?


I. Mr. Porat - Data is unlike oil in its generation and replenishment.
IL Mr. Rajan – The concern for the sustainability of economic decisions by firms.
Ill. Mr. Moynihan - Support of unconsolidated banking system with more liberal regulation.
(a) Only l (b) I and II (c) Only Ill (d) II and Ill

29. What were Raghuram Rajan’s views regarding the cheap and free services provided by firms?
(a) He wishes to have the big firms broken down, as in the U.S. in the 1930s.
(b) He regards the free services with caution as the firms must earn profits to exist.
(c) He expresses concern regarding the business of scale not being efficient.
(d) He wonders if the market competition will get fiercer in the future.

30. Why are ‘super star’ firms a cause of concern for the economy?
(a) They may become too big to control by political systems.
(b) They will topple governments if threatened.
(c) They pose concerns regarding security.
(d) They will drive in fiercer market competition.

Head Office: 127, Zone II, MP Nagar, Bhopal |+91-7676564400| https://www.toprankers.com Page 9 of 40
SECTION-B : CURRENT AFFAIRS, INCLUDING GENERAL KNOWLEDGE

Directions (Q.31–Q.65): Read the information given below and answer the questions based on it.

Passage(Q.31-Q.37): Negotiators for the Ethiopian government and regional forces from Tigray were due to
meet in the South African capital Pretoria for the first formal peace talks since war broke out two years ago. The
talks come after the Ethiopian military and their allies, who include troops from neighbouring Eritrea, captured
several large towns in Tigray, a region in northern Ethiopia, over the past week. The situation on the ground
appeared to put the government in a stronger position than its opponents going into the talks, though it was under
pressure from foreign powers, including the United States and European Union, to halt its offensive.

The war stems from a power struggle between the federal government and the authorities in Tigray, who
dominated a coalition that governed Ethiopia for almost three decades until they lost their grip on power in 2018.
The conflict has killed thousands, displaced millions, and left hundreds of thousands on the brink of famine. It
has also further destabilised the perennially volatile Horn of Africa region and complicated Ethiopia's diplomatic
relations with Western allies, who have been calling for a ceasefire.

31. Which of the following statement is correct about the Tigray War?
(a) The Tigray War is an ongoing civil war in the western part of the African Continent.
(b) The Ethiopian federal government is fighting on one side, and the TPLF and Eritrea are fighting on the
other.
(c) Both a and b
(d) None of the above

32. Which of the following statement is incorrect about the African Union?
(a) The African Union was announced in the Sirte Declaration on September 9, 1999, in Sirte, Libya.
(b) The AU's intention was to replace the Organisation of African Unity (OAU), which was founded in May
1963 in Addis Ababa by 32 signatory governments.
(c) The current chairperson of the African Union belongs to South Africa.
(d) None of the above.

33. Which of the following statement is correct about the Horn of Africa?
(a) The Horn of Africa (HoA), also referred to as the Ethiopian Peninsula.
(b) It runs along the Dead Sea's southern rim.
(c) The Horn of Africa Region includes the internationally recognised countries of Djibouti, Eritrea, Ethiopia,
and Somalia, as well as the de facto self-proclaimed Somaliland.
(d) Both a and b.

34. Who is the current Prime Minister of Ethiopia?


(a) Hailemariam Desalegn (b) Abiy Ahmed
(c) Meles Zenawi (d) Tamrat Layne

35. What is the correct expansion of TPLF?


(a) Tigray People's Liberal Federation
(b) Tigray’s Promotion of Liberation Foundation
(c) Tigray’s People Liberation Federation
(d) Tigray People's Liberation Front

36. The International Criminal Court is seated in which of the following country?
(a) Switzerland (b) USA (c) Netherlands (d) France

Head Office: 127, Zone II, MP Nagar, Bhopal |+91-7676564400| https://www.toprankers.com Page 10 of 40
37. Who is the current Minister of External Affairs of India?
(a) V. Muraleedharan (b) S. Jaishankar
(c) Meenakshi Lekhi (d) Smriti Irani

Passage(Q.38-Q.44): Reserve Bank of India (RBI) deputy governor ________ said there are many advantages
to the internationalisation of the rupee but, at the same time, there are risks associated with it which are
unavoidable if India wants to become an economic power.
In July this year, RBI came out with a scheme permitting the rupee settlement of international trade.
Internationalisation of the rupee is a process that involves increasing the use of the local currency in cross-border
transactions. It involves promoting the rupee for import and export trade and then other current account
transactions followed by its use in capital account transactions.

He spoke Thursday of the various advantages of internationalisation of the rupee but also alluded to the risks to
it. He said India is a capital-deficient country, and hence needs foreign capital to fund its growth.

“If a substantial portion of its trade is in rupee, non-residents would hold rupee balances in India which would
be used to acquire Indian assets. Large holdings of such financial assets could heighten vulnerability to external
shocks, managing which would necessitate more effective policy tools,” the deputy governor said.

38. Which of the following statement is correct about the Reserve Bank of India?
(a) The Reserve Bank of India is the nation's central bank in charge of regulating the Indian banking system.
(b) It is owned by the Ministry of Finance, Government of India.
(c) In accordance with the Reserve Bank of India Act, 1934, it was established in1935.
(d) All of the above

39. Which of the following statement is incorrect about Digital Rupee?


(a) RBI Digital Currency, also known as the digital rupee, is a digital version of the Indian rupee issued by the
RBI.
(b) It functions similarly to a sovereign currency and is exchangeable one-for-one with fiat currency.
(c) The RBI is contemplating three types of digital rupees: wholesale, retail, and general.
(d) Both a and c

40. RBI was established on the recommendation of ________________.


(a) Hilton Young Commission (b) Lyall Commission
(c) MacDonnell Commission (d) Fowler Commission

41. Which of the following body is responsible for printing and minting Indian currency?
(a) Bharatiya Reserve Bank Note Mudran Pvt. Ltd
(b) Security Printing and Minting Corporation of India
(c) Both a and b
(d) None of the above

42. Which deputy governor of RBI talked about internationalisation of the rupee?
(a) T Rabi Sankar (b) Mahesh Kumar Jain
(c) Michael Patra (d) M. Rajeshwar Rao

43. Which bonds are issued outside India but denominated in Indian Rupees, rather than the local currency.?
(a) Bharat bonds (b) Indian bonds
(c) Mumbai bonds (d) Masala bonds

Head Office: 127, Zone II, MP Nagar, Bhopal |+91-7676564400| https://www.toprankers.com Page 11 of 40
44. Shaktikanta Das is serving as _______ governor of India.
(a) 24th (b) 25th (c) 26th (d) 27th

Passage(Q.45-Q.51): Four years after it was placed on the “grey list” and penalised with severe financial
strictures by the Financial Action Task Force, Pakistan won a major reprieve on Friday, as the international
watchdog on terror financing and money laundering agreed to remove Pakistan’s name from the list of countries
under “increased monitoring”. Reacting to the decision, the Ministry of External Affairs said that Pakistan must
continue to take “credible, verifiable, irreversible and sustainable” action against terror groups on its soil.

In all, the FATF said Pakistan had completed two action plans comprising a 34-point task list in the period since
2018, and in a statement said that it “welcomes Pakistan’s significant progress” in its AML/CFT mechanisms.

“We are satisfied after a FATF inspection team went down, spoke to the authorities took a look, and verified,
and they were satisfied that there’s a high-level political commitment on the part of the Pakistani authorities to
not just implement the current set of action steps that they need to take, but they’re also committed to ongoing
reform,” said FATF President [1] from Singapore, speaking to the media at the end of the plenary session.

45. Which of the following statement is correct about the Financial Action Task Force?
(a) The Financial Action Task Force (FATF) is a joint initiative of G7 and the UN Security Council to develop
policies to combat terrorism financing and money laundering.
(b) FATF was established at the 1989 G7 Summit in Washington, D.C., and is headquartered there as well.
(c) Following the September 11th terror attacks, the organization's mandate was expanded to include terrorist
financing in 2001.
(d) Both a and b

46. Which of the following statement is incorrect about Group of Seven (G7)?
(a) The G7 is an intergovernmental political forum comprised of Australia, Canada, France, Germany, Japan,
the United Kingdom, and the United States.
(b) Its member nations are the world's largest developed economies and liberal democracies, as defined by the
IMF.
(c) Russia was a formal member of the group (as part of the Group of Eight) from 1997 to 2014.
(d) None of the above.

47. When did India become a member of the FATF?


(a) 2006 (b) 2008 (c) 2010 (d) 2012

48. Which of the following neighbour country of India was recently moved to the ‘black list’ of FATF?
(a) China (b) Myanmar (c) Nepal (d) Sri Lanka

49. Which of the following statement is correct about the Gulf Cooperation Council?
(a) The Gulf Cooperation Council (GCC) comprises Bahrain, Kuwait, Oman, Qatar, Saudi Arabia, and the
United Arab Emirates
(b) The main headquarters of the council is in Riyadh, Saudi Arabia
(c) The GCC's current member states are all monarchies
(d) All of the above

50. Who is the current Chief Economist of the International Monetary Fund?
(a) Gita Gopinath (b) Kristalina Georgieva
(c) Pierre-Olivier Gourinchas (d) Surjit Bhalla

Head Office: 127, Zone II, MP Nagar, Bhopal |+91-7676564400| https://www.toprankers.com Page 12 of 40
51. Who is the current President of FATF? Fill in the blank 1 by choosing the correct option:
(a) T. Raja Kumar (b) Marcus Pleyer
(c) Marshall Billingslea (d) Santiago Otamendi

Passage(Q.52-Q.58): Australia’s Bureau of Meteorology (BOM) predicted on August 25, 2022, that a third
consecutive event of La Nina could be underway, leading to additional rainfall along its east coast this summer.
The BOM predicted a 70 percent chance of the La Nina event. However, the United States and UK weather
agencies have claimed that the phenomenon is already underway.

Scientists said La Nina occurring in 2022, after having taken place in 2020 and 2021, would be a rare event. “We
are seeing an extended period of La Nina this year. It is the first time that this has happened since the 1950s
when the event started to be recorded. The years 1973-76 and 1998-2001 were consecutive La Nina years,”
Raghu Murtugudde, professor, department of atmospheric and oceanic science, University of Maryland, United
States, told Down To Earth.

La Nina is a climate phenomenon that takes place due to the cooling of surface ocean water on the tropical west
coast of South America. It can be described as a counterpart to El Nino, which involves the warming of ocean
currents in the equatorial Pacific Ocean. The combined phases of La Nina and El Nino are termed El Nino-
Southern Oscillation (ENSO) that affect rainfall patterns, global atmospheric circulation, and atmospheric
pressure across the planet.

52. Which of the following statement is incorrect about the geographical mechanism of La Nina?
(a) La Nina is caused by a build-up of cooler-than-normal waters in the tropical Pacific, which is defined as
the area of the Pacific Ocean between the Tropics of Cancer and Capricorn.
(b) This cold water is brought to the surface through a process known as upwelling, which is caused by
unusually strong, eastward-moving trade winds and ocean currents.
(c) Both (a) and (b)
(d) None of the above

53. Which of the following statement is correct about El Nino?


(a) El Nino is a part of the larger phenomenon known as the El Nino-Southern Oscillation, and it is its “warm
phase” (ENSO).
(b) El Nino affects ocean temperatures, ocean current speed and strength, coastal fisheries health, and local
weather from Australia to South Africa and beyond.
(c) El Nino events occur at regular intervals of two to seven years. As a result, El Nino is predictable in the
same way that ocean tides are.
(d) All of the above

54. Which of the following statement is incorrect about Sitrang?


(a) In October 2022, Cyclone Sitrang struck Myanmar and Bangladesh.
(b) It formed offshore the Andaman and Nicobar Islands and gradually intensified to become a high-end
cyclonic storm before making landfall over Bangladesh near Barisal.
(c) Sitrang, named by Thailand, is the first tropical cyclone of the 2022 post-monsoon season.
(d) None of the above

55. The concept of Nationally Determined Contributions (NDCs) is embodied in which of the following international
treaty?
(a) UN Framework Convention on Climate Change
(b) Paris Climate Accords
(c) Kyoto Protocol
(d) Copenhagen Accord

Head Office: 127, Zone II, MP Nagar, Bhopal |+91-7676564400| https://www.toprankers.com Page 13 of 40
56. Where will the 2022 United Nations Climate Change Conference conference be held?
(a) Geneva (b) Shubra El Kheima
(c) Sharm El Sheikh (d) Glasgow

57. The Sustainable Development Goals (SDGs) were proposed and set up by which of the following international
body?
(a) UN Economic and Social Council
(b) UN General Assembly
(c) United Nations Environment Programme
(d) Intergovernmental Panel on Climate Change

58. The India Meteorological Department (IMD) is an agency of which of the following ministry of the government
of India?
(a) Ministry of Earth Sciences
(b) Ministry of Environment, Forest and Climate Change.
(c) Prime Minister’s Office.
(d) None of the above

Passage(Q.59-Q.65): The Centre on Wednesday withdrew the long-awaited Personal Data Protection (PDP)
Bill, 2019 to replace it with a new bill with a ‘comprehensive framework’ and ‘contemporary digital privacy
laws’.

The PDP bill was first introduced in Lok Sabha on December 11, 2019. The bill was referred to Joint
Parliamentary Committee, which tabled its report in Lok Sabha on December 16, 2021. The committee proposed
a single law for dealing with both personal and non-personal datasets. The report was also under dispute as it
suggested moving towards complete localisation of data.

The withdrawn Bill had proposed restrictions on the use of personal data without the explicit consent of citizens.
It had also sought to provide the government with powers to give exemptions to its probe agencies from the
provisions of the Act, a move that was strongly opposed by the opposition MPs who had filed their dissent notes.

A statement containing the reasons for the withdrawal was circulated to the members of Lok Sabha. Reportedly
the statement included that the government was working on a comprehensive legal framework considering 81
amendments and 12 recommendations proposed by the JPC.

After the Bill was withdrawn, the Minister of State for IT tweeted that this will soon be replaced by a
comprehensive framework of global standard laws including digital privacy laws for contemporary and future
challenges, and catalyses Prime Minister Narendra Modi's vision.

59. Consider the following statements about The Information Technology (Intermediary Guidelines and Digital
Media Ethics Code) Rules, 2021.
I. Social media intermediaries, with registered users in India above a notified threshold, have been classified
as significant social media intermediaries (SSMIs).
II. SSMIs are required to observe certain additional due diligence such as appointing certain personnel for
compliance, enabling identification of the first originator of the information on its platform under certain
conditions, and deploying technology-based measures on a best-effort basis to identify certain types of
content.
Which of the abovementioned statements is false?
(a) Only I (b) Only II (c) Both I and II (d) None of the above

Head Office: 127, Zone II, MP Nagar, Bhopal |+91-7676564400| https://www.toprankers.com Page 14 of 40
60. A ___________ grievance redressal mechanism with varying levels of self-regulation has been prescribed for
publishers.
(a) Three tier (b) Two-tier (c) Five tier (d) Four tier

61. In which year was the Information Technology Act, 2000 (IT Act) amended to provide an exemption to
intermediaries from liability for any third-party information?
(a) 2009 (b) 2001 (c) 2008 (d) 2022

62. When were the Information Technology (Intermediary Guidelines and Digital Media Ethics Code) Rules, 2021
notified?
(a) February 25, 2021 (b) February 25, 2019
(c) January 26, 2020 (d) March 31, 2022

63. Social media companies with more than _______ registered users were to be considered ‘significant social media
intermediaries’.
(a) 5 lakhs (b) 10 lakhs (c) 50 lakhs (d) 100 lakhs

64. Personal Data Protection (PDP) Bill, 2019 proposed restrictions on______________?
(a) Celebrity endorsements
(b) The use of personal data without the explicit consent of citizens
(c) marketing activities of social media platforms
(d) None of the above

65. Who is the current IT Minister?


(a) Ashwini Vaishnaw (b) Nitin Gadkari
(c) Samyak Jain (d) Nikhil Aradhe

Head Office: 127, Zone II, MP Nagar, Bhopal |+91-7676564400| https://www.toprankers.com Page 15 of 40
SECTION - C: LEGAL REASONING

Directions(Q.66-Q.105): Read the comprehension and answer the questions:

Passage (Q.66 – Q.70): Marriage as an institution gives rise to a relationship between two partners. This
relationship also gives birth to different sets rights and obligations. These rights and obligations cumulatively
constitute ‘Conjugal rights’ and can be termed as essence of the marital union. The term ‘Conjugal Rights’ in
literal sense means ‘Right to stay together’. The same has its origins in feudal English Law, which at that time
considered a wife to be the chattel of the husband and not from any religious obligations or personal laws.
The section 9 of the Hindu Marriage Act and under section 22 of the Special Marriage Act provide that when
either the husband or the wife has, without reasonable excuse, withdrawn from the society of the other, the
aggrieved party may apply, by petition to the district court, for restitution of conjugal rights and the court, on
being satisfied of the truth of the statements made in such petition and that there is no legal ground why the
application should not be granted, may decree restitution of conjugal rights accordingly.
In T. Saritha Vengata Subbiah v. State, the court had ruled that that S.9 of Hindu Marriage Act relating to
restitution of conjugal rights as unconstitutional because this decree clearly snatches the privacy of wife by
compelling her to live with her husband against her wish. In Harvinder Kaur v. Harminder Singh, the judiciary
again went back to its original approach and held Section 9 of Hindu Marriage Act as completely valid. The ratio
of this case was upheld by the court in Saroj Rani v. S.K. Chadha.
In our country every citizens have a fundamental right to associate with anyone according to his/her wish, by the
matrimonial remedy of restitution of conjugal rights is freedom is violated as a wife is compelled to have an
association against her will, with her husband. In Huhhram v. Misri Bai the court passed the restitution against
the will of the wife. In this case though the wife had clearly stated that she would not wish to live with her
husband, still the court went ahead and gave the judgment in favour of the husband. This the primary shortcoming
or regressive application of this archaic law.
66. In which of the following grounds has the section 9 of the Hindu Marriage Act has been criticized or challenged:
I. Restitution of conjugal rights is an archaic law enacted as a result of patriarchy.
II. Inhibiting from exercising one’s sexual autonomy, the section is violative of right to privacy conferred by
article 21 of the Constitution.
III. Being forced to live with the other spouse violates freedom to reside in any part of the country and the
freedom to pursue profession of one’s choice.
IV The remedy of restitution to a spouse cannot be drawn from the personal laws of any religion in the country.
Choose the correct option:
(a) I and II (b) IV and II (c) III only (d) I, II and IV

67. Leanord and Penny have been in a marital tie for 8 years. All of a sudden Penny feels that she does not want to
be shackled by the marital bonds and pursue her abandoned acting career. So, she leaves their house to live all
by herself in Bombay for finding her inner self. After she left, Leanord made an application under section 9 of
the Hindu Marriage Act for restitution of conjugal rights. Penny has contested the petition contending that she
has a valid reason behind leaving and she no more fits in the institution of marriage. The matter is pending before
the district court for judgment. Choose the correct option.
(a) The court shall not allow the application as law cannot force Penny to stay with Leanord in derogation of her
right to privacy which ensures her sexual autonomy.
(b) The court shall allow the application as the decision of separation in an institution of marriage is bound to
be a mutual one as allowed in the cases of divorce.
(c) The court shall not allow the application as realizing that one no more fits into the institution and leaving to
explore one’s inner self is a reasonable excuse to separate from spouse.
(d) The court shall allow the application as Penny separated from Leanord without any reasonable excuse which
warrants strict order of restitution of conjugal rights.

Head Office: 127, Zone II, MP Nagar, Bhopal |+91-7676564400| https://www.toprankers.com Page 16 of 40
68. Continuing the above facts, if Penny has come back from Bombay after her failed acting stint yet again and
instead of moving in with Leanord, she is living with her boyfriend Kurt. Leanord makes an application as in the
above question, for an order to force her to move in with him. He refuses to initiate a divorce petition and desires
to live with Penny for life. In the view point of the author and the tone of the passage, decide whether Penny can
be forced by the court to stay with Leanord.
(a) Penny can be forced to go stay with Leanord as he is her husband and the law prohibits separating from him
unless for a reasonable cause.
(b) Penny cannot be forced to stay with Leanord, and ordering the same would be an infringement of her sexual
autonomy which is a protected right under right to life and personal liberty.
(c) Penny can be forced to stay with Leanord as separating for enjoying the company of boyfriend is a reasonable
cause to abandon one’s husband.
(d) Penny cannot be forced to stay with Leanord, as she is entitled to choose the person she wants to live with,
and if this idea is abhorred by Leanord he is free to apply for a divorce.

69. Jasraj and Farhana married each other under Special Marriage Act. Soon after marriage, Jasraj flew to abroad
for a business trip and to Farhana’s surprise Jasraj informed her through a call that he plans on settling there
forever. Farhana made an application under section 22 of the Special Marriage Act. In absence of any
representation from Jasraj, the court agreed with the contentions of Farhana that Jasraj left after the marriage and
has thus breached the institution of marriage, and thus passed an ex-parte order for restitution of conjugal rights
of Farhana. In an appeal, Jasraj’s lawyer argued that Jasraj strongly refrains from settling with Farhana and
therefore shall not be forced to do so. Decide whether the appeal be allowed or not.
(a) The appeal shall not be allowed as separation from a spouse unless for the reason of divorce is not permitted
by law.
(b) The appeal shall not be allowed as forcing Jasraj to stay with Farhana amounts to infringement of his right
to privacy ensured under article 21 of the Constitution.
(c) The appeal shall not be allowed because there is no legal basis for not allowing the application and thus the
court is thus entitled to pass an order for restitution.
(d) The appeal shall be allowed as not wanting to stay with one’s spouse is a reasonable excuse for separation
from them and discourages any order of restitution of conjugal rights by the court.

70. Jake and Amy married as a conclusion of love affair. However, after a few months of their marriage Amy
separated from Jake for his being an obsessive and interfering husband causing her immense mental cruelty.
Amy did not file for a divorce wondering that months of separation might enlighten Jake and he will be a more
liberal and understanding husband. Jake, on the other hand, opted for the legal process to force Amy to live with
him. The court has reserved the order for the application made under section 9 of the Hindu Marriage Act. Decide
whether the application shall be allowed or not.
(a) Application shall be allowed as marriage is a pious institution which cannot be shunned at whim of a person
without any particular reason.
(b) Application shall not be allowed as the court shall adopt a more liberal interpretation of the legal letter to
make orders in the interest of both the spouses without causing prejudice to either.
(c) Application shall be allowed because there is no reasonable excuse at Amy’s end for justifying her separation
from Jake and thus restitution shall be ordered.
(d) Application shall not be allowed as Amy has a reasonable excuse of strengthening her position in the marital
tie by abandoning Jake for some time.

Head Office: 127, Zone II, MP Nagar, Bhopal |+91-7676564400| https://www.toprankers.com Page 17 of 40
Passage (Q.71 – Q.76): In India the law addressing child labour is known as the Child Labour (Prohibition and
Regulation) Amendment Act, 2016. This law regulates the employment of children and does not allow children
below the age of 14 to work except as a child artist and in a family business. The Constitution of India, 1950
under Article 21(A) mandates free and compulsory education for all children in the age group of 6-14 years.
Article 24 also specifically prohibits the employment of children below the age of fourteen years in dangerous
factories which may cause them physical as well as long term mental harm.

The child labour law in India classifies children below the age of 18 into 2 categories, child (below 14 years)
and adolescents (between 14-18 years). The work which is allowed under the law depends on which age category
the child falls in. It is illegal for any person to employ or allow children under the age of 14 in any form of
occupation except as a child artist, or in a family business. A child as well as adolescent is allowed to work in a
family business to support the family, but only after school hours and during vacations.

For adolescents between the ages of 14-18, the law strictly prohibits them from working in a particular list of
hazardous non-industrial and industrial processes, such as mines or places which use inflammable substances or
explosives, firework shops, slaughter houses, food processing industry, etc.

Under the child labour law in India, any person who employs a child or an adolescent in any labour work will
be punished. Punishment for employing a child is imprisonment between six months and two years and/or a fine
between Rs. 20,000 and Rs. 50,000 and for employing an adolescent in any illegal occupation is imprisonment
for a period between six months and two years and/or fine between Rs. 20,000 and Rs. 50,000 and imprisonment
between one to three years if a person continues with child labour after having been punished once.

71. Kriti was a 17 year old girl living with her mother and grandmother. She had the sole responsibility to support
them because of the sudden demise of her father in an accident. After this incident, she was left as the only
earning member of her family. She started doing multiple shifts in a call centre to afford basic necessities for her
family and pay her own school fees. However, the income generated from this was not enough for the family.
She soon found a new well paid job in a firework industry, where she just had to monitor the works of the
labourers and report to the supervisor every evening. As soon as the authorities got to know about Kriti, they
rescued her and charged the supervisor for employing a child in the firework industry. Whether the supervisor
is guilty for employing a child or not?
(a) The supervisor is not guilty because the job in the firework industry was just a part time job for Kriti and she
worked full time only in a call-center.
(b) The supervisor is not guilty because the job for which Kriti was employed was only to monitor the work of
other laborers, which is non-hazardous in nature, though the industry is of hazardous nature.
(c) The supervisor is guilty because he employed a 17 year old girl in an industry, who should be focused on
her studies at this age instead of working in a hazardous industry.
(d) The supervisor is guilty because he employed Kriti, an adolescent in a firework industry which is hazardous
in nature and is impermissible under the law.

72. Jahnavi was an extremely talented dancer from a very small age. Her parents belonged to a poor community and
therefore they couldn’t pay for her dance classes. One of the members of Nritya Sangam, another community of
their village proposed that if they allow her to perform in the functions organized by them, he will pay for
Jahnavi’s dance classes and help them to meet their expenses. Another member of Jahnavi’s community was not
happy with this arrangement and therefore filed a complaint against the member of the Nritya Sangam
community under the Child Labour (Prohibition and Regulation) Amendment Act, 2016. Will this person be
punished as Jahnavi was only 11 years old at the time?
(a) No, the person should not be punished as the law permits any person to employ or allow children under the
age of 14 to work as a child artist.

Head Office: 127, Zone II, MP Nagar, Bhopal |+91-7676564400| https://www.toprankers.com Page 18 of 40
(b) Yes, the person should be punished as he should have helped her to learn dance and waited for her to turn
14 till he engages her to perform in the functions organized by their community.
(c) Yes, the person should be punished as it is an offence under the Child Labour Act to engage a child below
the age of 14 in any kind of work.
(d) No, the person should not be punished as he was only trying to help Jahnavi and her family. Moreover,
because of this arrangement it was possible for her to learn dance and hone her skills.

73. Akshay was a poor orphan child. He used to beg for food on the railway station in Palampur. One day, Rahim
saw Akshay on the platform begging for food and crying. He brought Akshay with him to his shop in Mirzapur
and trained him in repairing punctures. By the time Akshay turned 12, he has completely learned how to repair
bicycles, bikes and punctures and so he started working with Rahim at his shop. Rahim also offered him food
and some wage for a day’s work. This was reported by a nearby shop owner to the authorities, who charged
Rahim under the Child Labour (Prohibition and Regulation) Amendment Act, 2016. Decide if Rahim should be
held guilty in the court or not?
(a) Rahim should not be held guilty because he taught Akshay how to repair bicycles, bikes and punctures which
does not qualify as child labour and therefore he shall not be charged under the Child Labour (Prohibition
and Regulation) Amendment Act, 2016.
(b) Rahim should be held guilty because he employed Akshay, a child with age less than 12 years, in his puncture
repair shop. Such employment is not permissible under the law unless it is a family business or the child
works as an artist.
(c) Rahim should be held guilty because he made Akshay work all day and although he helped him to learn the
job and gave him food, he didn't send him to school.
(d) Rahim should not be held guilty because he brought Akshay, an orphan, to his home. He trained him in a
skill and also treated him like a son. Therefore, working at the puncture repair shop of Rahim was similar as
working in a family business for Akshay, which is permissible under the law.

74. Neelesh was a multi-millionaire industrialist and he wanted his son, Prasoon to join their family corporation
which deals in pre-shipment inspection services. However, his son had no experience of working anywhere
before so with an intention to make his 11 year old son earn some practical experience, he sent him to work at
his friend’s restaurant. Someone filed a complaint against the owner of the restaurant for hiring a child. On this
complaint regarding a child employed in the restaurant, authorities arrested the owner of the restaurant and
charged him under Child Labour (Prohibition and Regulation) Amendment Act, 2016. Decide the appropriate
punishment for Neelesh and/or the owner of restaurant.
(a) Neelesh should be punished with the imprisonment of 1 year and a fine of Rs 40,000 instead of the owner of
the restaurant because Neelesh was the one who allowed Prasoon to work at his friend’s restaurant.
(b) Both of them should be punished with the imprisonment of 2 years and a fine of Rs. 70,000 as Neelesh was
the one to allow Prasoon to work in a restaurant and the owner was the one to employ him in his restaurant.
(c) The owner of the restaurant should be punished with the imprisonment of 1 year and a fine of Rs 40,000
because he employed an 11 year old child in his restaurant.
(d) Neither of them should be punished as Prasoon was not employed as a labour but only joined the work at the
restaurant to gain practical experience to finally join his family business.

75. Abhay was 12 years old when his father lost his eyes in an unfortunate event. His family was very poor and the
income generated from making bangles was the only source of earning money for them to meet their daily ends.
Abhay was a very intelligent and hardworking student with a bright future. But when he started skipping school
frequently his teacher got worried about him and decided to visit his house to check on him. His teacher found
out that Abhay was working in the house to make bangles in place of his father to help his family meet the
demand of sales during the wedding season. His teacher complained to the relevant authorities against his family.
Decide if Abhay’s parents should be punished for involving him in their family business or not?
(a) Abhay’s parents should be punished for involving him in their family business because he should be allowed
to work in a family business to support the family only after school hours.

Head Office: 127, Zone II, MP Nagar, Bhopal |+91-7676564400| https://www.toprankers.com Page 19 of 40
(b) Abhay’s parents should not be punished for involving him in their family business because even a child
below the age of 14 can work in a family business to support his family.
(c) Abhay’s parents should not be punished for involving him in their family business because due to his father’s
accident he was the only earning member left in the family and he has to skip school to meet his family’s
needs.
(d) Abhay’s parents should not be punished for involving him in their family business because making bangles
is not hazardous work and it is permissible to work in such an industry under the law.

76. Assertion (A): It is legal for any person to employ or allow children under the age of 14 in any form of occupation
except as a child artist, or in a family business.
Reasoning (R): Under the child labour law in India, any person who employs a child or an adolescent in any
labour work will be punished.
(a) A is incorrect, R is correct.
(b) A & R both are correct, and R is the correct explanation for A.
(c) A & R both are correct, but R is not the correct explanation for A.
(d) A is incorrect, R is incorrect.

Passage (Q.77 – Q.81): General Defences, as the phrase indicates, are like weapons in the hands of the
defendants (also called ‘tortfeasor’) which help them evade liability. One of such defence is Volenti Non Fit
Injuria. This phrase derived from Latin translates to: “to a willing person, injury is not done,” In simple words,
it states that when a person voluntarily consents to a risk of injury, being aware of the consequences of it, then
that person cannot bring a claim against the other party, for any injuries suffered. Consent under the Law of Torts
has two elements involved in it. They are: knowledge of the party and voluntary agreement to the risks.
Therefore, if both of the above elements are present in a Tort, then the defence of volenti non fit injuria can be
claimed. But it must be kept in mind that such consent is not valid (a) If it is given under any kind of
misconception and accused knows about it (b) If consent is given while intoxicated (c) If given by a person of
unsound mind (d) Consent is given by a person under 12 years of age who does not have the maturity to
understand the consequences of the act. However, this defence has exceptions: (i) Carelessness of the other party:
you consent to the risks of the game but not the carelessness of the other party. (ii) However what you must
appreciate is, under the Law of Torts, a rescuer is not considered to have voluntarily accepted to the risks.
In this case, your actions do not contain your consent. Therefore, your claim will stand not only because you
wanted to rescue the child but also for the negligence of the stadium authorities for not maintaining the stadium
properly. However, it is also seen that sometimes a person doesn’t need to rescue, and yet the person acts in
rescue thereby claiming the exception of the defence. Then his claim will not succeed. (iii) Illegal acts: To claim
the defence of volenti non fit injuria, the act committed by the defendant must not be illegal or unlawful.

77. X learned that Y had lately purchased a large amount of diamond jewellery in preparation for her daughter's
wedding the following month. She planned to transfer it in the locker the following morning. Since X was already
aware of this, he chose to attack the same night. Y's servant caught him and started screaming as he was ready
to escape with the jewellery. Y and her husband entered the room. As a result, X pointed a revolver at Y's husband
and threatened to shoot him if they did not let him escape with the valuables. Because Y had already contacted
the police, she was unconcerned about what would happen and informed X that she didn't care because the police
were already on their way. When he heard this, X became enraged and shot Y's husband. X claimed volenti
non fit injuria when Y reported him to the police. Decide.
(a) X can assert volenti non fit injuria because he had previously told Y of the possibility of shooting, to which
she consented.
(b) X cannot assert volenti non fit injuria since the necessary conditions are not met in this circumstance.
(c) X can plead volenti non fit injuria because Y was aware that X would undoubtedly shoot her husband.
(d) X cannot contend volenti non fit injuria since Y consented under threat of harm.

Head Office: 127, Zone II, MP Nagar, Bhopal |+91-7676564400| https://www.toprankers.com Page 20 of 40
78. N's friend K asked him for a book to which he may refer throughout his exams. N agreed to give the book because
they were in different schools and had exams on different days. K promised to hand over the book before the
exam because he just wanted to take a few notes from it. K was referring to his exam, which was scheduled after
N's exam, but N mistook K for referring to N's exam. When N asked for the book two days before his exam, K
responded that he meant he would give the book before his exam, which is in two weeks. Even though K was
aware that N had granted his assent based on a misconception of the facts, he did not return the book, leading N
to be unable to take proper notes and hence score lower in the exam. When N threatened to file a complaint
against K, he responded that he would claim volenti non fit injuria. Analyze.
(a) K can assert volenti non fit injuria because he had already mentioned that he would return the book before
his exams.
(b) K cannot assert volenti non fit injuria since the case at hand is an exception to the defence of volenti non fit
injuria.
(c) K can plead volenti non fit injuria because N was aware that K was about to return the book before K's exam.
(d) K can claim volenti non fit injuria since the necessary conditions are met in the present circumstance.
79. Choose the option that you consider closely matches the information in the passage.
(a) A person cannot sue the other party for any damages incurred if they unknowingly accept a risk of injury
while being aware of the implications of doing so.
(b) It should be emphasized that consent under volenti non fit injuria is not valid if it is given out of fear or
misunderstanding and the accused is unaware of it.
(c) Under Tort Law, a rescuer is not regarded to have knowingly accepted the dangers. The actions in this case
do not indicate consent.
(d) The plaintiff's acts must not be unlawful or improper in order to raise the defence of volenti non fit injuria.
80. R let H use his microwave so she could make a cake for her daughter's birthday. She brought the microwave
home for a week with R's knowledge and approval because R wouldn't be in town. She neglected to monitor the
temperature while using the microwave and turned it on while it was empty, causing internal damage to the
appliance. When R learned of this, she requested H to cover the cost of the repairs. H then asserted that she had
taken the microwave with R's permission. She can therefore argue volenti non fit injuria. What do you think in
light of the fundamentals of volenti non fit injuria?
(a) R authorized H to use the microwave; therefore she can't demand payment because she was aware of the
dangers.
(b) H did not intend to damage the microwave, so R cannot sue for damages because it was an unforeseeable
accident.
(c) R may file a claim for damages because she authorized the use of the microwave and not the damage.
(d) R is entitled to compensation and may bring a lawsuit against her for negligence that caused the microwave
damage.
81. T was in her room watching a movie. Nobody was home that day, so she had to prepare dinner for herself. She
began watching the movie while accidentally leaving the gas knob on. Her room's door was locked, so she was
unable to smell anything. She left her room to close the windows when it suddenly started to rain, and as she
turned around to go back inside, she noticed her kitchen on fire. She attempted to contact someone, but the
network was poor because of the persistent rain. When he saw her house on fire, her neighbour L, who had just
returned from a party, immediately sought to save her. As a result, he suffered severe burns. When his parents
requested T to cover the medical expenses, she asserted volenti non fit injuria. Make an informed decision and
adopt an appropriate course of action.
(a) T never requested L's assistance and he consented to his own injuries while being aware of the fire's risks,
thus T shouldn't be responsible for paying the compensation.
(b) T should provide compensation since the aforementioned scenario qualifies for the volenti non fit injuria
exception.
(c) T shouldn't compensate out because she sustained similar injuries in the fire and nobody was to blame.
(d) Given that she requested assistance and is now expected to return that assistance, T should provide
compensation.
Head Office: 127, Zone II, MP Nagar, Bhopal |+91-7676564400| https://www.toprankers.com Page 21 of 40
Passage (Q.82 – Q.86): Negligence in legal terms means the breach of a duty of care that result in damage. The
following three ways make the constituents for negligence: a legal duty to exercise due care on the part of the
party complained of towards the party complaining the former’s conduct within the scope of duty; breach of the
same duty and consequential damages. Medical negligence is a combination of two words. The second word
solely describes the meaning, though the meaning of negligence has not been described in a proper way but it is
an act recklessly done by a person resulting in foreseeable damages to the other. Negligence is an offense under
tort, IPC, Indian Contracts Act, Consumer Protection Act and many more. Medicine is such a profession where
a practitioner is supposed to have requisite knowledge and skill needed for the purpose and has a duty to exercise
reasonable duty of care while dealing with the patient. The standard of the care depends upon the nature of the
profession. A surgeon or anesthetist will be determined by the standard skill of average practitioner in that field
while in case of specialists, a higher skill is needed. If the doctor or a specialist doesn’t attend a patient admitted
in emergency or under his surveillance and the patient dies or becomes victim of consequences which could have
been avoided with due care from the doctor, the doctor can be held liable under medical negligence. Moreover,
the liability of the doctor cannot be invoked now and then and he can’t be held liable just because something has
gone wrong. For fastening the liability, very high degree of such negligence is required to be proved. Res Ipsa
Loquitur means ‘things speak for itself’. While deciding the liability of the doctor it has to be well established
that the negligence pointed out should be a breach in due care which an ordinary practitioner would have been
able to keep.

82. X experienced an extremely severe stomach infection. He tried a number of medications, but the ache got worse
every day. When he eventually visited a doctor, he was told that he would need to have a surgery so that the
cause of his pain could be removed. The medication that would have kept his blood pressure at the proper level
throughout surgery was not given to him by the doctor. Fortunately, he had already had a healthy blood pressure
level, which helped the surgery go smoothly. The family of X sued the doctor after learning about his negligence.
What are your thoughts on the situation at hand?
(a) Since the doctor failed to provide the patient the necessary medication, which could have led to further harm,
he should be held accountable for his negligence.
(b) Since the doctor failed to inform X of the change in blood pressure level before to the surgery, he should be
made responsible.
(c) Considering that X was obligated to inform the doctor of his blood pressure fluctuation before the procedure,
the doctor shouldn't be held accountable.
(d) Since the prerequisites for holding the doctor liable under the negligence tort are not satisfied in this instance,
the doctor should not be made liable.

83. Choose the one that, in your opinion, best fits the passage's facts and the assertion it contains.
(a) The doctor can also be held liable even if something minor goes wrong or have his liability brought up
sometimes. A very high degree of such negligence is not required to be demonstrated in order to establish
the culpability.
(b) There is a legal obligation for the party being complained of to exert reasonable care for the party
complaining about the former's conduct within the limits of the duty. A breach of the same duty and
subsequent damages are necessary to hold a person accountable.
(c) Medicine is one of those professions where the practitioner must have the information and skills necessary
for the job in addition to having an obligation to treat patients with a decent amount of care. The nature of
the profession is irrelevant to the level of care required.
(d) Though it hasn't been properly defined, negligence is defined as an action performed irresponsibly by one
person that causes another person to suffer an unanticipated harm.

Head Office: 127, Zone II, MP Nagar, Bhopal |+91-7676564400| https://www.toprankers.com Page 22 of 40
84. Y went to the dentist for some consultation. Since one of his teeth had been damaged by bacteria and the infection
was worsening, he was supposed to have it removed. In order to prevent the germs from spreading while
performing the treatment, the doctor was required to administer an antibiotic. The doctor applied a foot cream
rather than the tooth cream. His throat and neck began to become infected as a result. Y then complained about
the doctor in court. Provide your insights on Y's course of action.
(a) Y should refrain from filing a lawsuit against the doctor because it was a genuine error on the doctor's part
and there was no intention to do so.
(b) Since the doctor failed to exercise reasonable care while providing the treatment, which resulted in additional
damage, Y should hold the doctor accountable.
(c) Since the infection was so severe that it may have spread even in the absence of the doctor's negligence, Y
shouldn't hold the doctor accountable.
(d) To get his infection cured, Y should ask the doctor for a refund of the expenses as well as compensation.

85. Z had the practice of having frequent medical checks since his employer paid for them each month for all of the
employees. Despite the fact that Z had left the company, he persisted with the habit since he believed it would
help him take better care of his health. The clinic intern discovered him diagnosed with a specific allergy while
Z was receiving his check-up. He informed the doctor as a result, before he conducted any additional testing.
The doctor overlooked to take notice of this, and at his check-up the following month, he gave him some vitamins
that aggravated his allergy and made him sicker altogether. Z consequently filed a medical malpractice lawsuit
against the intern. Is the lawsuit valid?
(a) Z will be successful in suing the doctor since the doctor failed to review the diagnosis report, which caused
Z's allergy to worsen.
(b) Z's claim will be unsuccessful since he should have sued the doctor instead of the intern.
(c) Because the intern did his part of the job and took reasonable care to alert the doctor, Z will not be successful
in pursuing legal action.
(d) Z will be successful in suing since the intern was supposed to check whether the doctor was aware of Z's
allergy while administering the supplements.

86. X had a massively important meeting set at 7 p.m. in Delhi, so he needed to go there by boarding a flight at 2
p.m. He was going to sign a contract that would have earned him ten times as much profit as he had earned the
previous year. He heard someone crying in the garden close to his apartment as he was leaving. While a
child screamed for assistance, he discovered a man lying unconscious. Since he didn't want to miss his flight, he
called his friend, a doctor, and requested him to come right away. He also gave the daughter his friend's contact
information. The man passed away because his friend got stuck in traffic and couldn't arrive in time. The daughter
then filed a medical negligence claim against X. Comment.
(a) Since there was no duty breach from X's side, he is not supposed to be held accountable.
(b) Given that his inaction contributed to the man's death, X should be held liable.
(c) Since X had already requested his friend to reach as soon as possible and had also provided the child the
pertinent information, he shouldn't be held responsible.
(d) Since the man passed away as a result of the doctor’s late arrival, he needs to be held accountable.

Head Office: 127, Zone II, MP Nagar, Bhopal |+91-7676564400| https://www.toprankers.com Page 23 of 40
Passage (Q.87 – Q.91): Damnum Sine Injuria is a legal maxim which refers to damages without injury or
damages in which there is no infringement of any legal right, which is vested with the plaintiff. Since no legal
right has been infringed, no action lies in the cases of Damnum Sine Injuria. It is an implied principle in law that
there are no remedies for any moral wrongs, unless and until any legal right has been infringed. Even if the act
or omission done by the defendant was intentional, the court will not grant any damages to the plaintiff. So, it
can be rightly said that an act which is lawfully or legally done, without negligence and in the exercise of a legal
right, such damages as comes to another thereby is damage without injury whereas Injuria Sine Damnum is a
violation of a legal right without causing any harm, loss or damage to the plaintiff and whenever any legal right
is infringed, the person in whom the right is vested is entitled to bring an action. So in total, Injuria Sine Damnum
refers to the remedies which are provided in the form of damages or compensation in violation of any legal right,
such that if the legal right is violated then an action lies even if there is no harm to another. In other words, it is
an infringement of a right where no loss is suffered but it creates a cause of action. The conclusion of the two
maxims are such that one is a moral wrong for which the law gives no remedy even though they cause great loss
or detriment to the plaintiff’s but on the other hand other one is a legal wrong for which the law does give a legal
remedy though there be a violation of a private right, without actual loss or detriment in that particular case.
87. Given that there was no bakery in the entire area, X opened one in the H district. He had a monopoly on the
business. He charged more than what was going on in the market in other districts. One G moved into the same
district and chose to open a bakery because he had 10 years of experience in the industry. He used higher-quality
materials than X and priced 10% less for all items as compared to X. As a result, demand for X's items
plummeted, and he incurred a significant loss. As a result, he filed a lawsuit against G for monetary damages. G
claimed Damnum Sine Injuria as a defence.
(a) G's claim is valid since the requirements of Damnum Sine Injuria are met in this case.
(b) G's claim is invalid since the circumstances in the given case do not satisfy the prerequisites of Damnum
Sine Injuria.
(c) G's claim is invalid because G violated X’s right to freedom of trade and profession.
(d) G's argument is convincing because he has the freedom to follow any profession he wants.
88. T was a leading figure in fashion. She intended to do just one video out of the two because her followers had
consistently asked her to do one on both her skincare routine and diet plan. To allow her followers decide, she
posted a poll on her story. R was ready to cast her ballot when her Feedgram suddenly stopped working,
preventing her from doing so. As a result, T's diet plan received the most votes, whereas R preferred it to be a
skincare regimen. Feedgram was sued by her for violating her right to vote. Comment on R's actions and the
measures she took.
(a) R is entitled to compensation from Feedgram since the error of the application resulted in a violation of her
right to vote.
(b) R is not eligible for compensation because Feedgram was not at fault for the app's discontinuation since it
was a result of her excessive use of the app.
(c) Since neither the conditions of injuria sine damnum nor damnum sine injuria are met in the particular
situation, R is not qualified to receive compensation from Feedgram.
(d) R is entitled to compensation from Feedgram because the app's technical defect prevented her from casting
a vote, which prevented her from streaming the video of her choice.
89. M submitted applications to six schools, with XYZ School being her top preference. During the admissions
process, she went to XYZ School to enroll in standard 11. There, she was rejected admission for no apparent
reason. She was soon contacted by PJ School, which had accepted her application and offered her admission.
Despite being admitted to a school, she chose to submit a complaint against XYZ School for depriving her of
her right to education without justifiable grounds. What are your thoughts on the given instance?
(a) Since M was finally admitted to PJ School, she is not allowed to complain about XYZ School.
(b) M is not eligible to complain about XYZ School since they are not obligated to let her know why she was
turned down for admission.
(c) M has the right to sue XYZ School under the provisions of the maxim of Injuria Sine Damnum.
(d) M has the right to sue XYZ School since she was denied admission without being given a valid explanation.
Head Office: 127, Zone II, MP Nagar, Bhopal |+91-7676564400| https://www.toprankers.com Page 24 of 40
90. N worked for a multinational corporation that paid him more than he anticipated. He was also permitted to take
two leaves each month at the discretion of the employer, as stated in the appointment letter guidelines. Because
the fiscal year ended in March, he was instructed by his employer not to take any leave for the month since there
was a lot of work outstanding. He was also instructed to report to the office on Saturday and execute the assigned
tasks. The identical instructions were conveyed to the other employees. N, who was dissatisfied, filed a complaint
against his employer for infringing his right granted to him at the time of his employment. Discuss.
(a) Since his legal right to take a leave off was infringed in the aforementioned situation, N may sue his
employer.
(b) N may bring a claim against his employer because the latter was required to notify him earlier that no leave
would be permitted.
(c) N cannot sue the employer because his legal rights were not violated in the circumstances of the case at hand.
(d) N is not entitled to sue his employer because the company's laws do not correspond to national laws.

91. Select the response that, in light of the information in the passage, is the least erroneous.
(a) A legal dictum known as Injuria Sine Damnun refers to damages without harm or damages in which the
plaintiff's legal rights are not violated.
(b) Unless and until a fundamental right has been violated, there are no legal remedies for moral wrongs. Even
if the defendant intentionally committed the act or omission, the court will not award the plaintiff any
damages.
(c) It is accurate to say that harm without injury results from an act that is done properly or legally, without
negligence, and in the exercise of a legal right.
(d) When a legal right is violated without causing the plaintiff any injury, loss, or damage, this is known as
Damnum Sine Injuria, and the person who owns the legal right has the right to file a lawsuit.

Passage (Q.92 – Q.96): Product liability law finds its origin in the common law concept of caveat venditor,
meaning "let the seller beware", placing the onus on the seller for any problem that the buyer might encounter
with a service or product. Product liability implies the responsibility of a manufacturer or vendor of goods to
compensate for injury caused by defective merchandise that it has provided for sale. Product-liability cases have
consequently led to development in general principles of contract law and tort law; wherein in contract law,
product liability is based on the principle of 'warranty', and in tort law product liability is based on the principles
of 'negligence' and 'strict liability'.

In most cases, intentional or negligent action is required for liability to attach. However, in some cases (including
certain product liability actions), liability can attach even though the defendant did not engage in negligent or
intentionally harmful behaviour. Manufacturing defects occur when certain items are built incorrectly or with
missing parts due to flaws or neglect throughout the manufacturing process. Only a few products are affected,
and not all of them.

Products liability refers to the liability of any or all parties along the chain of manufacture of any product
for damage caused by that product. This includes the manufacturer of component parts (at the top of the chain),
an assembling manufacturer, the wholesaler, and the retail store owner (at the bottom of the chain). Products
containing inherent defects that cause harm to a consumer (or someone to whom the product was loaned, given,
etc.) of the product would be the subjects of products liability suits. While products are generally thought of as
tangible personal property, products liability has stretched that definition to include intangibles (i.e. gas), naturals
(i.e. pets), real estate (i.e. house), and writings (i.e. navigational charts). Products liability is derived mainly from
torts law.

Head Office: 127, Zone II, MP Nagar, Bhopal |+91-7676564400| https://www.toprankers.com Page 25 of 40
92. Hari has wanted to colour his hair for quite some time. It was his birthday, and he went to Harpreet's saloon-
style edge, where Harpreet treated Hari's hair with a dye and as a result Hari contracted dermafitis. The
manufacturers of the dye, M & Kinsley Company had delivered the dye to Harpreet in labelled bottles together
with a small brochure of instructions. Both the label and the brochure contained a warning that the dye might be
dangerous to certain skins and a test of skin was recommended before it was used. Hari sues the manufacturers
M & Kinsley Company for negligence for product liability. Would the manufacturers of the dye be held liable
to Hari?
(a) No, but Harpreet can file a damage suit against the manufacturers of the dye, M & Kinsley Company.
(b) No, as Products contained inherent defects that has caused harm to a Hari.
(c) No, as the liability of Harpreet comes in the picture for his negligence.
(d) No, as there is no manufacturing defect that has caused harm to the consumer.

93. D, a doctor, inoculate some children with Hepatitis B vaccine. The children were taken ill because of the presence
of some toxin in the vaccine used by D who purchased it from C, a reputed chemist. C, in turn, had purchased
the vaccine from M, the manufacturer. Decide the liability, if any, of C and M towards the children.
(a) All of them including D, will be held liable based on the principle of negligence.
(b) M and then C will be held liable in order under product liability suit.
(c) C and then M will be held liable in order under product liability suit.
(d) Both C and D will be held liable as Products containing inherent defects that cause harm to a consumer of
the product would be the subjects of products liability suits.

94. Yashasvi purchased a pair of blue jeans from 'Lovely Outfitters' at Ansal Plaza for his girlfriend Girija. Girija
developed acute dermatitis as a result of wearing it due to the presence of extra sulphites, which were discovered
to have been irresponsibly left in the denim manufacturing process. The denim had been purchased from a cloth
mill and therefore the defendant, the manufacturer of the jeans, did not know of the defect in the cloth. In a
damages complaint brought by Girija, the defendant claims, among other things, that he did nothing to prevent
examination before the trousers reached the real user, and that he did not sell the garment to Girija in any
case. Is defendant’s plea maintainable?
(a) Yes, as it should be the buyer who should been more careful while purchasing the item.
(b) No, as the buyer was prevented from conducting the examination of the cloth and thus defendant’s plea is
not maintainable.
(c) Yes, as the seller did not prevent the examination before the trousers reached the real user.
(d) No, as the Products containing inherent defects that cause harm to a consumer of the product would be the
subjects of products liability suits.

95. Z purchased a pullover for her husband, H, from an outfitters based in Kamla Park, Mumbai. After wearing it,
H suffered acute dermatitis due to the presence of extra sulphites that had been negligently left in the
manufacturing of the wool. The wool was not manufactured by the defendant's pullover manufacturer; it was
purchased from another manufacturer, and hence the defendant was unaware of the flaw in the wool. In an action
for damages brought by H, the defendant pleads inter alia, that he had done nothing to prevent examination
before the garment reached the actual user and in case he did not sell the garment to H. Make your case for H.
(a) The product disputed will be made a subject of product liability suit.
(b) H had acted negligently as he failed to examine the product before buying it from the outfitter.
(c) The Product contained inherent defects that harmed H and thus the manufacturers should be held liable.
(d) H will lose the case since the suit for damages in the current case is against the manufacturer from whom
defendants purchased.

Head Office: 127, Zone II, MP Nagar, Bhopal |+91-7676564400| https://www.toprankers.com Page 26 of 40
96. From the following illustration given below, choose the one the in which a consumer can file a product liability
claim based on manufacturing defects:
I. While manufacturing a set of pencils, the lead fails to be put in one of the pencils.
II. A new type of car is being created to be suitable for desert sands but the tyre of the car was designed
negligently such that those tyres are sinking into sand.
III. The seller while selling an iron did not make the consumer aware that a particular thing if not removed could
increase its temperature uncontrollably fast.
IV. A consumer buys a washing machine with an express warranty of good functioning for 12 years but the
machine malfunctions within 6 years.
(a) I (b) II and III (c) I and II (d) All of the above

Direction (Q.97 – Q.101): Mistake of fact arises when an accused misunderstands some fact that negates an
element of crime. The legal maxim, ignorantia facti excusati ignorantia juris non excusat means ignorance of
fact is an excuse, but ignorance of law is no excuse. Mistake of fact will not be a valid defense if the act
committed is illegal itself. Section 79 states that nothing is an offence which is done by any person who is
justified by law, or who by reason of a mistake of fact and not by reason of a mistake of law in good faith,
believes him to be justified by law, in doing it. The legal presumption is that everyone knows the law of the land.
An act will not be an offense if it is committed in a manner by a person who by mistake of fact believes to be
bound by law. However, mistake of law per se is not excusable. Section 81 states that nothing is an offence
merely by reason of it being done with the knowledge that it is likely to cause harm, if it be done without any
criminal intention to cause harm, and in good faith for the purpose of preventing or avoiding greater harm to
person or property. It is important to note that insanity plays a vital role in criminal law and its legal sense is to
be understood rather than medical sense in order to claim defense under IPC. Section 84 of the Indian Penal
Code says that nothing is any offence which is done by a person who is of unsound mind at the time of doing
the act and is incapable of knowing the nature and consequences of the act and that what he is doing is wrong or
contrary to law.

97. R was a notorious criminal in the J District. He was just recently found guilty of murder. K, the superintendent
of police, was tasked with detaining R before sundown. He left towards R's residence to arrest him as soon he
obtained the arrest warrant. On the other hand, R escaped as soon as he learned that a police officer was heading
his way to make an arrest because he had already heard from his sources that this was the case. Instead of R, he
arrested B, who was his twin brother and looked just like him. Determine the liability.
(a) As stated in the passage, K should be held accountable for his actions because he violated the law by arresting
the wrong person, which is not acceptable.
(b) K should be held accountable because when he detained B, he had some criminal intent.
(c) Since K has a defense available to him under section 79 of the Indian Penal Code, he should not be made
accountable.
(d) K cannot claim mistake of fact because the act he committed was unlawful in nature.

98. Y experienced a significant mental illness. He would occasionally behave normally and other times not. Since
he had a spacious garden and loved children, many kids used to play there. One day, he made the decision to
organize a children's picnic in the garden where they could play games and eat delicious snacks. On the same
day, the neighbors discovered two kids unconscious and seriously injured in the garden. Although Y wasn't there,
it was later discovered that he attempted to flee the scene while hiding the weapons beneath the ground. Their
parents are suing him for murder. Specify his responsibility.
(a) Since Section 84 of the Indian Penal Code allows for the defense of insanity, Y shall not be held responsible
for his actions.
(b) Section 81, which provides that nothing is an offence if it is done without any criminal intent to do harm,
can be used by Y as a defense.

Head Office: 127, Zone II, MP Nagar, Bhopal |+91-7676564400| https://www.toprankers.com Page 27 of 40
(c) Y is permitted to use the mistake of fact as a defense under section 79 because it occurs when an accused
person misinterprets a fact that disproves a crime's element.
(d) Y cannot assert the defense of insanity since the conditions that necessitate it have not been met.

99. J was rushing to get to his office on time because he had a crucial customer meeting to attend. He was due to get
at his office by 10am, but it was then 9:45 and he was still 30 minutes away. He would receive a raise if he could
work out the deal with the client. Therefore, he moved against the flow of traffic, did not follow the traffic lights
and drove pretty quickly. The traffic cops pulled him over at the next stop and seized his license. J claims mistake
of fact. Decide.
(a) J may assert a defense of fact given that he misunderstood the direction of traffic and accidentally disregarded
traffic signals in an effort to make it to work on time.
(b) J cannot claim that he made a mistake of fact because breaching the rules cannot excuse him from the offence.
He must be aware of them and obey them.
(c) J cannot use mistake of fact as an excuse because his carelessness could have resulted in someone losing
their life.
(d) Since the prerequisites have been met, J may invoke the defense provided by Section 79 of the Indian Penal
Code.

100. According to the information in the passage, pick the answer that is the most accurate.
(a) Nothing constitutes a crime just by virtue of being carried out in good faith with the desire to prevent or avert
greater injury to people or property, even if it is done in the knowledge that it will probably result in harm.
(b) It is essential to remember that insanity plays a significant part in constitutional law and that in order to assert
defense under IPC, its legal sense rather than its medical sense must be considered.
(c) If a person commits an act in a way that leads them to genuinely think they are under legal obligation, it will
be considered a crime.
(d) Ignorance of fact is an unacceptable defense, but it is not acceptable to be ignorant of the law, according to
the legal proverb ignorantia facti excusati ignorantia juris non excusat.

101. P and his friends went to an amusement park to enjoy the ride. By 11am, all of the friends were scheduled to
assemble at the main entrance. P made the decision to wait within the park and enjoy some ice cream because
his friends were snarled up in traffic. He entered without purchasing the ticket because the ticket checker was
not present at the counter. The guard saw that he didn't have the ticket-checking stamp on his hand when he was
leaving with his friends. As a result, the guard assessed him a fine of 5000 rupees. Speculate about the validity
of the accusation.
(a) P will not be subject to any penalties because the ticket checker had an obligation to ensure that everyone
had a ticket before entering the park.
(b) P won't face any penalties since he can argue that he didn't realize getting into the park required buying a
ticket, hence he made a factual error.
(c) Due to P's mistake, which the passage says has no defense, he will be held accountable.
(d) P will be held accountable because he had the opportunity to pay the ticket price when he left the park rather
than just opting not to do so.

Passage (Q.102 – Q.105): When, at the desire of the promisor, the promisee or any other person has done or
abstained from doing, or does or abstains from doing, or promises to do or to abstain from doing, something,
such act or abstinence or promise is called a consideration for the promise. If what is given in exchange for the
promise has value in the eyes of the law, the court will not question whether that value is adequate and will not
interfere with the fairness of the bargain made by the parties. The law on this point, regarding the adequacy of
consideration is simple and clear. The consideration is not required being of a particular fixed value or an
approximation to the promise for which it is exchanged but it must be have some value in the eyes of the law. It
must change the promisee‘s position after the consideration is acted upon or transferred from the promisee to the
promisor. Consideration is indispensable for making an agreement to be enforceable as per Section 10 of the

Head Office: 127, Zone II, MP Nagar, Bhopal |+91-7676564400| https://www.toprankers.com Page 28 of 40
ICA. Section 25(1) provides that when an agreement without consideration is expressed in writing and is made
on account of natural love and affection between the parties, the same is valid. On the contrary, the existence of
the near relation between the parties is not sufficient to show that there is affection between them. As natural
love and affection cannot be assumed every time when no other motive is known, the court presumes by
considering the relation between the parties. Section 25(2) states that there is no requirement of consideration, if
a person does an act or gives his service voluntarily to the promisor without the knowledge of the promisor and
in turn, the promisor shows his willingness to compensate for it by an undertaking. It was stated by the Court
that the term ‘voluntarily done’ shows the performance of something based on one‘s own impulses, will and
choices. There should be no constraint or pressure or suggestion from anyone.

102. H had a wide range of storybooks. He made the decision to sell all of them to his friends for 100 rupees each.
Later, he learned that each book was originally priced 600 rupees. His parents rebuked him severely because his
recklessness cost them a significant amount of money. He realized that if he had charged the appropriate amount,
he would have made a substantial profit. He thus requested his friends to pay the balance, but they turned him
down. For breach of contract, H filed a lawsuit, determine the feasible possibility.
(a) H is entitled to make a claim for the remaining amount because he was supposed to receive consideration
which is equal to the cost of the books.
(b) Since it was his responsibility to charge the correct price in the first place, he is not entitled to the remaining
sum.
(c) H cannot claim the balance amount because the validity of consideration is what counts rather than the
sufficiency of the same.
(d) H cannot claim the balance amount since he did not purchase the books; he received them for free and thus
is not entitled to claim any amount.

103. Choose the statement that corresponds to the passage as per your understanding of the same.
(a) The Court declared that the phrase voluntarily done denotes action taken in accordance with one's own
desires, will, and decisions. There shouldn't be any restrictions, pressure, or recommendations from anyone.
(b) If a person does an act for the promisor or renders his services with the promisor's knowledge and the
promisor then expresses his readiness to make good on that act by making an undertaking, no consideration
is necessary.
(c) After the consideration is produced or transferred from the promisee to the promisor, it must not alter the
promisee's position. An agreement must include consideration in order to be enforceable under Section 10
of the ICA.
(d) The consideration has to be of a certain set value or equivalent to the promise for which it is traded and it
must have some legal worth.

104. K and F were first cousins yet they did not share a very close bond. K moved to Dubai to complete his graduation.
He was returning to town to celebrate Christmas with his family. On the request of F, K brought him a tonne of
clothing from Dubai. They had a conversation about the same over an Email. He gave F the garments and
requested payment in the same amount. F refused to pay the money, stating that the clothes were given to him
as a gift out of love and affection, so he was not required to make a payment. Decide the validity of his claim.
(a) F is absolutely correct when he asserts that an act performed by a promisor out of love and affection does
not constitute a requirement for consideration.
(b) Since F did not specifically request the garments that K brought in accordance with his own wishes, he is
not obligated to pay the price for them.
(c) F must pay for the clothes K bought for him because Section 25(1)'s requirements are not being met.
(d) If K refuses to provide the garments to F for free, F shall not accept the clothes from K if he does not wish
to pay for them.

Head Office: 127, Zone II, MP Nagar, Bhopal |+91-7676564400| https://www.toprankers.com Page 29 of 40
105. V threatened to keep M from leaving his house unless M gave him his brand-new McLaren. Since he was afraid,
M initially offered V his car at no cost but then eventually requested payment for it. According to V, M gave
him his car out of his own free will; hence as per Section 25(2), it did not require any sort of consideration.
Decide.
(a) V shall not be held accountable because Section 25(2) provides that no consideration is required if a person
performs an act freely for the promisor.
(b) Since the requirements under section 25(2) are not met, V will be held responsible for paying the
consideration for the car.
(c) Since V gave the car to M out of love and affection, he will not be held responsible for paying the car's
expenses.
(d) Since M cannot take such a valuable car out of V's possession without paying the amount, M will be held
responsible for paying V the charges for the vehicle.

Head Office: 127, Zone II, MP Nagar, Bhopal |+91-7676564400| https://www.toprankers.com Page 30 of 40
SECTION - D: LOGICAL REASONING

Directions (Q.106-Q.135): Read the passage given below and answer the questions that follow-

Passage (Q.106-Q.110): Signifying a vital shift in US policy on marijuana, President Joe Biden recently
pardoned around 6,500 people who were convicted of simple possession of marijuana under federal laws and
were serving time in jails. While the pardon paves the way for these people to smoothly reintegrate into the
society and pursue gainful activities, including work and education, what is more significant is the President’s
promise of looking into the possibility of separating marijuana from the legal category of dangerous semi-
synthetic drugs like heroin, Ecstasy and LSD. This revisiting of the decades-old rule is remarkable, for it sets
right a wrong by addressing the long-brewing tussle between culture and policing over the recreational use of
substances. Central to the clash is the prevalence of cannabis as a traditionally grown and consumed plant by
many communities and its growing acceptance worldwide.

India should take the lead from the US and step up efforts to decriminalise marijuana. This will help reduce the
dependence of addicts on such lethal drugs as chitta that are destroying lakhs of lives and families in the region
and, in turn, aid the states in their ongoing uphill battles against the menace of drugs. Excluding marijuana from
the purview of the harsh Narcotic Drugs and Psychotropic Substances Act-1985 would be in alignment with the
stance that India took at the UN in 2020, when, along with the majority of the nations, it voted for removing
cannabis from the list of most dangerous substances, like the deadly and addictive opioids.

Pushed for by advocates, around 50 countries now allow medicinal cannabis programmes. Encouragingly, in the
past few years, some states of India too have climbed down from the draconian stand on the matter. Uttarakhand
has regularised cannabis cultivation, while Gujarat has removed bhang from the list of intoxicating drugs; UP
and MP have similar policies. Earlier this year, Himachal Pradesh cleared the decks for cannabis farming for the
medicinal value and industrial use of hemp. A balanced approach to keeping up the fight against psychotropic
substances, drug trafficking and narco-terror and realising the benefits of traditional, organic alternatives is the
need of the hour.

106. As per the passage, in which of the following cases, would the likelihood of being pardoned by the US president
for convicted people decrease?
i. If they had been convicted for possessing substances like marijuana, used for medicinal purposes.
ii. If they had been convicted for possessing semi-synthetic drugs like heroin and LSD.
iii. If they had been convicted for offences like committing first degree murder and extortion.
(a) Only i and iii (b) Only ii and iii (c) Only ii (d) All i, ii and iii

107. Which of the following opinions can be attributed to the author of the passage?
(a) Recreational use of all kinds of substances should be permitted.
(b) The US is generally the first country to bring in changes to society.
(c) Marijuana should be excluded from the purview of drug laws in India.
(d) If the US is doing something, others should follow its path.

108. The author’s arguments would be most weakened by which of the following pieces of evidence, if true?
(a) The ban on Marijuana has no practical implications, and people keep consuming it.
(b) Marijuana has been known to cause significant lung damage after prolonged consumption.
(c) Marijuana is a known pain reliever in cancer treatments.
(d) Marijuana helps in preventing drug and alcohol addiction.

Head Office: 127, Zone II, MP Nagar, Bhopal |+91-7676564400| https://www.toprankers.com Page 31 of 40
109. Which option best summarises the author’s arguments in the passage?
(a) Since the USA has signalled a policy shift on marijuana, India must follow it impetuously.
(b) It is about time India legalises organic drugs, especially when countries like the USA have started doing so.
(c) Marijuana is part of the culture, there is no need to put a blanket ban on it; restrictions should end.
(d) The revisiting of the decades-old rule in the USA is remarkable as it paves the way for the countries to control
marijuana.

110. Which among the following is a statement of fact rather than a statement of opinion?
(a) Pushed for by advocates, around 50 countries now allow medicinal cannabis programmes, India should allow
it too.
(b) Encouragingly, some states of India have climbed down from the draconian stand on the matter of marijuana.
(c) Excluding marijuana from the purview of the drug laws in India would be in alignment with the stance that
it took at the UN in 2020.
(d) The pardon by the President paves the way for the convicts to smoothly reintegrate into the society and
pursue gainful activities, including work and education.

Passage (Q.111-Q.115): A face recognition attendance system has been put in place in schools in Andhra
Pradesh. This policy change is aimed at raising the quality of education and improving the learning experiences
of students, including their assessment of the formative level.

Several reports have hailed this move as a revolutionary shift that will help children develop an array of abilities
and provide them with experiences that will enhance their problem-solving and critical-thinking faculties. This
is nothing new in this. Teacher absenteeism got recognised as a systemic issue almost four decades ago and
became a policy goal soon after that. Keeping teachers in their schools has become an end in itself and the
policies framed to ensure this have an inspectorial character rather than a reformatory one.

So far, the behaviourist schemes of policymakers were restricted to students, But it seems now that teachers too
are being targeted. Reward and punishment postings have, of course, been part of the system for long. However,
what is new is the link between the policy measure and its intended result. A teacher — who has to be
“motivated” and offered “incentives” to download an app — is expected to build intellectual abilities and make
learning inquiry-driven. How marking a teacher’s face on an app will lead to the acquisition of intellectual
abilities by his/her students remains a puzzle.

Issues related to teachers’ absenteeism cannot be resolved by an app. They require a humanist understanding of
teachers as professionals, their predicaments and needs. Sudden visits by school inspectors, community
supervision and even panchayat scrutiny were part of educational policies of the past three decades. Like the
app-based attendance recording system, these surveillance mechanisms failed to appreciate that teaching is
fundamentally a social interaction. It is about students and teachers putting their minds together.

A teacher’s presence is, of course, necessary but it’s more important that he/she has curiosity and passes on the
spirit of inquiry to students. The Reports of the National Commission on Teachers (1983-85), NCF-2005 and the
NCFTE (2009) have much that’s useful for this purpose. All three policy documents argued for treating teachers
as professionals and not as caged employees whose presence has to be constantly monitored. Two decades ago,
UNESCO described the attitude of policymakers towards teachers as a silent crisis by asking, Where Have All
the Teachers Gone?

Head Office: 127, Zone II, MP Nagar, Bhopal |+91-7676564400| https://www.toprankers.com Page 32 of 40
111. Which of the following is the main idea of the passage that the author is trying to convey?
(a) There are several reasons that point out that the surveillance system on teachers would be only partly
successful.
(b) Resolving teacher absenteeism needs a more humanist approach, not a surveillance system.
(c) Teacher absenteeism does not need resolution, and the policymakers are missing something more crucial.
(d) Giving too much importance to teacher absenteeism would likely aggravate the problem than solve it.

112. If the information in the passage is correct, then which of the following follows from the passage?
(a) The face recognition app might end up highlighting the absenteism crisis at the cost of massive investment.
(b) UNESCO has failed in its attempts to resolve the issue of teacher absenteeism.
(c) Teachers don’t play a pivotal role in schools; students can learn among themselves.
(d) The new surveillance system is a novel idea to encourage teachers to come to schools.

113. Which among the following has been assumed by the government in installing a face recognition system?
(a) The presence of teachers in a school is a must for the students to learn.
(b) The presence of teachers is not enough for the students to learn.
(c) The presence of teachers will inculcate values in children.
(d) Keeping teachers in their schools is paramount to preventing absenteeism in children.

114. By claiming that the teacher’s presence is necessary in the last paragraph, the author:
(a) Argues that the schools can’t run without the aid of the teachers.
(b) Highlights another issue that needs to be taken care of along with teacher absenteeism.
(c) Argues that teachers don’t play a pivotal role in aiding the learning process.
(d) Highly regards the teachers for their contribution to schools.

115. Which of the following best reflects the title of the passage?
(a) Face recognition attendance system
(b) Face recognition attendance system and its advantages.
(c) The defeated purpose of face recognition attendance system
(d) The rampant absenteeism amongst teaching staff.

Passage (Q.116-Q.119): When B.R. Ambedkar stood up to move the draft Constitution of India in the
Constituent Assembly, one might have expected an appeal to national sentiment, some rousing rhetoric or, at the
very least, an outline of the vision the Constitution was designed to serve. Instead, Ambedkar painstakingly
pointed out how the Constitution of India was both derived from, as well as distinct from, the Constitution of the
United States of America. This was a curiously humdrum speech by the erudite scholar. But in its content, it
captured the essence of law-making in India — the fundamental question that lawmakers in India usually tend
to ask is what an analogous law looks like in other countries rather than what problem the law seeks to solve in
India. Then they go about ‘adapting’ that base draft to ‘Indian conditions’. Today, seven decades on, the
phenomenon is both deeper and wider.

The homepage of e-Amrit, the governmental programme on electric mobility, has a section devoted to how
Norway, Chile, the United Kingdom, besides others, have achieved the adoption of electric vehicles. Sixty-three
pages in the lead opinion in the seminal judgement upholding the right to privacy survey judgments on privacy
from the UK, the US, Canada, South Africa besides other jurisdictions. These are unsurprising — research
methodology taught in law schools, public policy institutions and lawyers’ chambers stresses on the need to learn
and derive from international best practices while making and interpreting law in India.

The point here is not that Indian lawmakers and judges should be oblivious to what is going on in other parts of
the world. That is neither sensible nor desirable. The point, instead, is this: by using Western laws as starting

Head Office: 127, Zone II, MP Nagar, Bhopal |+91-7676564400| https://www.toprankers.com Page 33 of 40
points, lawmakers tend to miss out on the Indian cultural context in which the law is meant to operate, the State
capacity available to enforce such a law in India, and a deep understanding of what it will take for the law to be
successful here. Copying without appreciating these critical factors is often responsible for the law missing its
mark.

116. Which of the following options best summarises the author’s arguments in the passage?
(a) India should be oblivious to the laws going around in the world.
(b) Copying Western or global laws is not the best course for India.
(c) Adapting from international best practices is a symptom of our national character.
(d) India must make its own laws without prejudice due to other laws.

117. Which among the following could be attributed to the author’s opinion when B.R. Ambedkar gave an unexpected
speech on the draft Constitution?
(a) The author is surprised at how B.R. Ambedkar isn’t proud of the Constitution.
(b) The author readily agrees with the point of view of B.R. Ambedkar regarding the Constitution.
(c) The author considers B.R. Ambedkar to be a learned scholar, and Ambedkar’s opinions are his opinions.
(d) The author understood the context of framing of the Constitution that later entered other policy matters.

118. The information in the passage disallows which of the following to be inferred from the passage?
(a) India’s lawmakers are more concerned about framing original laws than borrowing from analogous ones.
(b) India’s judges and lawyers have no choice but to submit to what the analogous laws around the world say.
(c) Analogous laws are more problematic than what is originally perceived by the lawmakers.
(d) All of the above

119. What role could the following sentence play in the context of the passage? “Policy-makers need to carefully set
out the problem statement of what the law is trying to solve.”
(a) This statement would strengthen the author’s arguments in the passage.
(b) This statement is an assumption based on which the author’s argument depends.
(c) This statement could be an extension of the author’s claims based on the information in the passage.
(d) This statement weakens the arguments presented by the author in the passage.

Passage (Q.120-Q.124): Nobody wants to worry about where their next meal is coming from. Yet over 800
million people around the world - most of them in developing countries - suffer from chronic food deprivation
on a daily basis. Global hunger creates a cycle that people can't escape from. It causes individuals to be less
productive and more prone to disease, which in turn makes them less able to improve their livelihoods or earn a
better income. But with the planet producing enough food to feed everyone, why are so many people going
hungry? World hunger refers to pockets of the human population who regularly do not get enough food to eat.
A vast majority of the world's hungry people live in developing countries. The situation is worst in Africa, with
almost 21 per cent of the population there facing hunger on a daily basis. As a region, Sub-Saharan Africa
struggles the most, with nearly one in every four people facing undernourishment.

The effects of world hunger are as diverse as they are devastating. At its core, a diet characterized by an
insufficient intake of calories, proteins, vitamins and minerals impedes human development at every level - for
infants, children and adults. In turn, this negatively impacts the health, education, economic and social
development of entire communities across the globe. Hunger is more than just having enough food to eat - it's
about having enough nutritious food to eat. Poor nutrition is attributed to 45 per cent of deaths in children under
the age of five, while one out of every six children in developing countries is underweight. A significant portion
of the world's population is also affected by micronutrient deficiencies - a shortage of vitamins and minerals
necessary for good health. This is sometimes called "hidden hunger," because there may not be obvious, visible
signs of malnutrition. Iron deficiency in women of reproductive age is a prime example of a micronutrient
deficiency. The right to food is a basic human right. And while our planet produces enough food to feed the more

Head Office: 127, Zone II, MP Nagar, Bhopal |+91-7676564400| https://www.toprankers.com Page 34 of 40
than 7 billion people who inhabit it, systemic inequality and economic disparity have led to unbalanced
distribution and unequal access. The Food and Agriculture Organization of the United Nations estimates that
one-third of the food produced worldwide is lost or wasted. At the same time, hunger kills more people every
year than malaria, tuberculosis and AIDS combined. But this is an injustice that we can remedy in our lifetime -
if done right, agriculture, forestry and fisheries can provide enough nutritious food for all.

120. Which of the following statement(s) gives the Best Conclusion to the passage?
(a) The passage talks about the issue of world hunger, especially in developing countries and goes on to discuss
its causes and effects and what measures could be taken as a remedy.
(b) The passage talks about the issue of world hunger, especially in developing countries.
(c) The passage talks about the issue of world hunger, especially in developed countries
(d) The passage talks about the issue of world hunger, especially in developed countries and goes on to discuss
its causes and effects and what measures could be taken as a remedy.

121. Which of the following option(s) is furthering the passage's claims?


(a) The problem of hunger is not just limited to nutrient deficiency or deaths but, also has negative impacts on
the health, education, economic and social development of people.
(b) The remedy lies in erasing systemic inequality and economic disparity.
(c) Both A and B.
(d) Neither A nor B.

122. Which of the following option(s) is/are indicative of the author's arguments as mentioned in the passage?
(a) About 55 per cent of total deaths in children below the age of 5 can be attributed to Poor nutrition.
(b) Hunger is more than just having enough food to eat - it's about having enough nutritious food to eat.
(c) Our planet produces enough produce to feed about 7 Lakh people.
(d) Agriculture, forestry and fisheries can never provide enough nutritious food to satisfy the needs of people
across the world.

123. Which of the following statement(s) is not in agreement with the author?
(a) The Blue Revolution refers to the significant growth and intensification of global aquaculture production -
domestication and farming of fish, shellfish, and aquatic plants.
(b) The period between 1991 to 2003 is known as the period of the Golden Revolution in India. The Golden
revolution is related to the production of honey and horticulture.
(c) The Purple Revolution is the cultivation of Lavender as a new aroma crop, especially in the Kashmir
Himalayas.
(d) All of the above.

124. Which of the following statement(s) does not coincide with the reasonable understanding of the passage?
(a) South African region is the most undernourished region across the world.
(b) With a large population on the Earth to feed, issues such as hunger would always prevail.
(c) The problem is that global hunger creates a such cycle that people find it impossible to escape from.
(d) Distribution system of food produced could help tackle the issue to a great level.

Head Office: 127, Zone II, MP Nagar, Bhopal |+91-7676564400| https://www.toprankers.com Page 35 of 40
Passage (Q.125-Q.130): Immigration has increasingly been a defining factor in the Republican and Democratic
parties’ shift into their respective populist corners. In 2016, with the vilification of illegal and legal immigration
at the core of his campaign, Donald Trump tapped into the socio-cultural anxieties of the American electorate.
This mainly informed the defection of the working class (after having voted twice for Barack Obama) in favour
of Trump’s electoral prospects in crucial swing states. Thereafter, in the Left’s bid to wrest the mantle of being
the “worker’s party,” views once held by a small fraction of progressives re-emerged. One such view - once held
by the likes of Vermont Sen. Bernie Sanders and Ohio Sen. Sherrod Brown - was over temporary work visa
programmes driving down wages and putting locals out of jobs.

However, in a sign of his own turn to the Left on the matter, the Biden campaign’s promise on expanding
temporary work visas begins with the assertion: “High skilled temporary visas should not be used to
disincentivize recruiting workers already in the US for in-demand occupations.” Furthermore, Biden has
committed to “a wage-based allocation process” and “enforcement mechanisms to ensure they are aligned with
the labour market and not used to undermine wages.”

Biden’s inconsistency on the matter, however, could have been informed by the political pressures of the
election. For instance, appealing to progressives could have been the motivation for touting the cause of
correcting wage disparities and, thereby, protecting American and foreign workers alike. With this position’s
indirect apprehension over immigration, Biden could also deftly avoid his campaign from being “an uneasy fit”
with Democrats’ largely pro-immigration posturing against Trump’s anti-immigrant stance over the past four
years.

Beyond intra-party positioning, Biden’s stance could have also been informed by the inclination to not be
completely out-of-step with the rising nativism amongst the American electorate, as some polls earlier this year
reported an unprecedented 79 percent of Americans supporting “a temporary stop to all immigration.” This could
have been a key determinant, as it was around that time of increased apprehension over immigration when Trump
signed an executive order to pause (for 60 days) issuance of new permanent residencies or green cards.

From the standpoint of US-India ties, a Biden administration would likely be supportive of New Delhi’s desire
to not interlink the H1B matter to other divergences in the bilateral relationship. For instance, the Trump
administration had momentarily considered limiting H1B visas to 15 percent for “any country that does data
localization.” Whereas, on exercising continuity over some positive developments of the Trump years, New
Delhi would expect the Biden administration to at least continue negotiations over a “totalisation agreement,”
which would permit Indian professionals in the US to withdraw their social security deposits after their visas
expire.

However, on his promised expansion of the H1B visa programme, Biden will mostly focus on gradually reversing
Trump’s record in order to better gauge the political viability of increasing temporary work visas at a time of
economic uncertainty in the US.

125. “Immigration has increasingly been a defining factor in the Republican and Democratic parties’ shift into their
respective populist corners.” Which of the following statements, if true, would weaken the author’s argument?
(a) Indian Americans who are eligible to vote are already citizens, and hence immigration-related policies do
not affect them.
(b) Republican candidates of both the past and this election have not had any immigration-related issues in their
manifesto.
(c) Last year a poll was conducted, and a significant number of democrats, who generally tend to be pro-
immigration, were found to be anti-immigration.
(d) The challenges related to immigration are now prominent in the U.S, making the Republican and Democratic
parties choose a side: pro-immigration or anti-immigration.

Head Office: 127, Zone II, MP Nagar, Bhopal |+91-7676564400| https://www.toprankers.com Page 36 of 40
126. The author states that the working class who had earlier voted for Obama, voted for Trump, as he vilified legal
and illegal immigration. What assumption does the author base this statement on?
(a) The working-class votes on the basis of policies and not previous patterns of voting.
(b) Trump finds the majority of his voter base in the working class.
(c) The issue of immigration affects the working class the most.
(d) The working class became against Obama.

127. What, according to the author, is the reason behind Biden’s assertion that “High skilled temporary visas should
not be used to disincentivize recruiting workers already in the US for in-demand occupations.”?
(a) He wants to reassure the public that immigration would not take their jobs away.
(b) Biden believes that both the natives and the foreigners can co-exist in the economy without one taking the
others’ jobs.
(c) He wants to appease both the domestic and the foreign workers due to the imminent elections.
(d) Biden wants to garner the votes of the immigrant workers more than the residential workers.

128. Which of the following is not true about the passage?


(a) Immigration is an important factor in the voting pattern of American voters.
(b) Biden wants to protect both immigrants, and the native population, and hence is neither against nor extremely
pro-immigration.
(c) Under Biden’s regime, the grant of H1B visas would depend upon bilateral ties between India and the USA,
as he wants to protect both native and foreign interests.
(d) The working class sided with Trump’s anti-immigration stance.

129. Based on the contents of the passage, what would Biden’s logical course of action be regarding immigration?
(a) Biden would allow all immigrants to enter the USA.
(b) Biden would allow H1B holders to enter the USA and hold highly-skilled jobs.
(c) Biden would give more weightage to the local workers.
(d) Biden would impose wage thresholds to protect the native workers.

130. In the question below are given four statements followed by four conclusions. You have to take the given
statements to be true even if they seem to be at variance from commonly known facts. Read all the conclusions
and then decide which of the given conclusions logically follows from the given statements disregarding
commonly known facts.
Statements:
All threes are gloves.
All nights are gloves.
All barks are threes.
Some gloves are onions.
Conclusions:
I. Some onions are barks.
II. Some nights are barks.
III. All barks are gloves.
IV. Some threes are onions.
(a) Only I follows (b) Only III follows
(c) Only II, III and IV follow (d) Only I, III and IV follow

Head Office: 127, Zone II, MP Nagar, Bhopal |+91-7676564400| https://www.toprankers.com Page 37 of 40
Direction (Q.131-Q.135): Four families, each consisting of a married couple and one child, are waiting at a
pediatric clinic. Among them A, B, C and D are males whose ages are 28, 30, 32 and 34 years, not necessarily
in that order. P, Q, R and S are females, whose ages are 25, 26, 28 and 29 years, not necessarily in that order. W,
X, Y and Z are the children whose ages are 3, 4, 7 and 8 years, not necessarily in that order. The following
particulars are known about them.
(i) D’s child is not the youngest, but the youngest female’s child is the youngest.
(ii) X’s age is twice the difference of ages of X’s parents but his mother Q is younger to at least two other
ladies.
(iii) R’s husband is nine years elder to P. Neither P nor S is W’s mother.
(iv) Y is as much elder to Z as his mother is to A’s wife. P’s husband is six years elder to C’s wife.
(v) B and R is a married couple and their child is four years old.

131. How old is B’s wife?


(a) 25 years (b) 26 years (c) 28 years (d) 29 years

132. Who is Y’s father?


(a) D (b) B (c) C (d) A

133. Which of the following combinations is correct?


(a) B – R – Z (b) C – P – Y (c) D – Q – Y (d) C – Q – X

134. Who among the following is A’s child?


(a) W (b) X (c) Y (d) Z

135. How old is Q’s husband?


(a) 32 (b) 28 (c) 26 (d) 30

Head Office: 127, Zone II, MP Nagar, Bhopal |+91-7676564400| https://www.toprankers.com Page 38 of 40
SECTION - E: QUANTITATIVE TECHNIQUES

Direction (Q.136-Q.140): In an annual match, 1008 students participated in a College. The ratio of the number
of girls to the number of boys is 5:3 respectively. Out of the total girls, 20% participated in cricket and the
remaining girls participated in hockey, football, and tennis in the ratio of 2:3:4, Two–thirds of the total boys
participated in football and the remaining boys participated in hockey and cricket in the ratio of 4:5.

136. What is the approximate percentage of the boys who have participated in cricket out of the total number of boys?
(a) 18.5% (b) 23% (c) 16% (d) 27%

137. What is the approximate percentage of the girls who participated in hockey out of the total number of
participants?
(a) 11% (b) 15% (c) 6% (d) 20%

138. What is total number of girls who have participated in football and tennis together?
(a) 492 (b) 392 (c) 384 (d) 368

139. What is the ratio of number of boys to the number of girls who have participated in hockey?
(a) 1:2 (b) 2:1 (c) 4:3 (d) 3:2

140. What is the difference between the number of boys and that of girls who have participated in cricket?
(a) 63 (b) 35 (c) 56 (d) 126

Direction (Q141-Q.145): The following line-graph represents the number of branches of public sector banks
in India:
(In thousands)
90

80

70

60

50

40

30

20

10

0
1960-61 1970-71 1980-81 1990-91 2000-01

141. Find the percentage increase in the number of branches from 1960-61 to 1990-91
(a) 500% (b) 400% (c) 300% (d) None of these

Head Office: 127, Zone II, MP Nagar, Bhopal |+91-7676564400| https://www.toprankers.com Page 39 of 40
142. If the total number of public sector banks in 1960-61 and 2000-01 were 14 and 28 respectively, then find the
ratio of average branches per bank in these two years.
(a) 1:4 (b) 4:1 (c) 1:3 (d) 2:5

143. There- is a- proposal to merge SBI ‘and its associate banks in 2000-01 due to which the total number of branches
will decrease by 5,000. After this SBI has 40% of total branches.Find the percentage change in the total branches
of SBI its associate banks before and after merging.
2 2
(a) 16 3 % increase (b) 16 3 % decrease
2 2
(c) 14 % decrease (d) 14 % increase
7 7

144. If 40% of the total branches are in rural areas in 1970-71 and 60% of the total branches are in urban areas in
2000-01, then find the increase in the number of rural branches in these two yeas.
(a) 48000 (b) 40000 (c) 44000 (d) 42000

145. If there were 10,000 branches less in 1980-81, then between which two decades, is the growth rate in number of
branches the lowest?
(a) From 1990 − 91 to 2000 − 01 (b) From 1960 − 81 to 1990 − 91
(c) From 1960-61 to 1970-71 (d) None of these

Direction (Q.146-Q.150): A science institute offer M.Sc. with specialization in Mathematics, Physics and
Chemistry. 45% of total student in M.Sc. are girls. 30% of total number of boys in M.Sc. have admitted in
Mathematics. 40% of total girls are in Chemistry department. Ratio of number of boys and girls in Mathematics
department is 3:2. Out of total number of boys in institution, 50% are studying Physics. Total number of boys in
Mathematics is 297.

146. How many students are pursuing M.Sc.?


(a) 2000 (b) 2200 (c) 1800 (d) 1600

147. Difference between number of girls studying Chemistry and that in Mathematics is:
(a) 198 (b) 126 (c) 162 (d) 226

148. Number of boys studying Physics is what percent of total number of students in institution?
(a) 27.75% (b) 27% (c) 27.25% (d) 27.5%

149. What is the ratio between number of boys and girls studying Chemistry?
(a) 7:8 (b) 11:18 (c) 18:11 (d) 8:7

150. Total number of boys in the institution is what percent more than total number of girls in the institution?
1 2
(a) 22.5% (b) 19% (c) 33 3 % (d) 22 9 %

Head Office: 127, Zone II, MP Nagar, Bhopal |+91-7676564400| https://www.toprankers.com Page 40 of 40

You might also like